Этого треда уже нет.
Это копия, сохраненная 18 января 2020 года.

Скачать тред: только с превью, с превью и прикрепленными файлами.
Второй вариант может долго скачиваться. Файлы будут только в живых или недавно утонувших тредах. Подробнее

Если вам полезен архив М.Двача, пожертвуйте на оплату сервера.
Тред тупых вопросов №113 Cecilia edition 514000 В конец треда | Веб
Тред вопросов о жизни, Вселенной и всём таком.

Спрашиваем то, за что в других местах выдают путёвку в биореактор. Здесь анонимные учёные мирового уровня критически рассмотрят любые гениальные идеи и нарисованные в Paint схемы.

Предыдущий тут: >>510223 (OP)
https://2ch.hk/spc/res/510223.html (М)

Q: Можно быстрее?
A: Можно упасть в пузырь альбукерке, наса уже почти надула его.

Q: Я начитался охуительных историй про уфологию, че делать, нам жопа?
A: Да, тебе жопа, можешь сгонять в зогач или куда оттуда пошлют.

Q: Что будет с человеком в вакууме без скафандра / если он упадет на черную дыру / попробует ступить на поверхность газового гиганта/солнца?
A: Он умрёт.

Q: Почему бы не привязать ракету к воздушному шару или стартовать с горы?
A: Космос - это не как высоко, а как быстро, большая часть энергии ракеты уходит на разгон вбок.
Подробнее тут https://what-if.xkcd.com/58/ (английский) https://chtoes.li/orbital-speed/ (перевод)
2 514042
Школьникам прогуливающим уроки физики в школе

смотреть от

https://www.youtube.com/watch?v=XqCNajiaW3U

и до

https://www.youtube.com/watch?v=DrpDqSD8DJY
3 514047
>>14000 (OP)
Во первых ебаный стыд, абулькерка!
Во вторых это надувателтство.
В третьих нельзя допускать интерференции, иначи придется идти пешком.
4 514137
Меняется ли состав атмосферы с набором высоты, и если да то как
5 514154
>>14137
Растет число легких молекул (водорода и гелия). На самом верху атмосферы, на высотах выше 200 км уже очень горячо около 1500 градусов и под действием солнечной радиации молекулы начинают ионизироваться. Вообще там пиздец происходит, лучше туда не ходи.
6 514308
что мешает людям сейчас полететь на Марс? Слышал что лететь нужно всего 9 мес.
7 514315
>>14308
Технически - ничего принципиально не мешает. Просто посадить живого человека на Марс человечеству сейчас по силам.
Но не так уж сильно это человечеству нужно. А развлечение страшно дорогое.
UzacUAc08Yw.jpg241 Кб, 493x750
8 514328
>>14000 (OP)
Зачем тратить деньги на космонавтов, решать вопросы со снаряжением, припасами, запасами кислорода? Еще радиация, а это уже весомый повод, чтобы не рыпаться с родной планеты.
Если даже цель всего этого пилить бабос, то делать это более прагматично и с большей эффективностью можно за счет беспилотных космических летательных аппаратов.
Технологии уже сейчас позволяют собирать ресурсы на Луне и в поясе астероидов.
Ядерные космические двигатели изобретены, а вместо этого все мозгуют, как бы нам на Марс обосраться цель, красная пыльная пустыня высадиться.
9 514339
>>14328
изобретены не значит реализованы на практике
10 514348
Не нашел проектов по искусственным горам кроме курортной в Голандии. Почему не навали кучу земли скажем в 20-40-60 и т.д. км. и от туда стартовать ракетами или ещё чем нибудь. Нашел такие проекты как Космический фонтан, Пусковая петля, Космический мост да тот же лифт, но вот искусственную гору даже близко. На вики было просто упоминание, но нечего конкретного. Чё за фигня
11 514349
>>14348
См. ОП-пост:

> Q: Почему бы не привязать ракету к воздушному шару или стартовать с горы?


> A: Космос - это не как высоко, а как быстро, большая часть энергии ракеты уходит на разгон вбок.


> Подробнее тут https://what-if.xkcd.com/58/ (английский) https://chtoes.li/orbital-speed/ (перевод)

12 514350
Gavni iz /b
13 514351
>>14315
а разве тут можно решать с точки зрения выгоды и эффективности? Тут скорее другие критерии оценки. И лететь нужно только лишь потому что уже можно. А не потому что это все не окупится. Высадиться на другой планете это новый этап развития цивилизации, который навсегда войдет в историю Земли.
14 514356
>>14351

> а разве тут можно решать с точки зрения выгоды и эффективности?


Как видишь, можно: именно так обычно решают адекватные руководители (всякие там президенты стран и проч.).
Выгода бывает не только грубо материальная. Многое в космосе было сделано ради престижа, например. В долговременных научных целях. И так далее.
Но просто для того, чтоб порадовался анон с двача, огроменные средства и силы вбухивать не будут.
15 514373
Как чёрная дура может сохранять заряд (пусть и краткое время)? Ведь информация о наличии заряда не сможет выйти из-под горизонта. Или это связано с тем, что для внешнего наблюдателя вещество под горизонт не проваливается, а застывает очень близко к нему, и поэтому может ещё быть источником поля?
16 514375
Кхм блеат
1.jpg55 Кб, 1300x650
17 514388
>>14000 (OP)
Это не она про Звезду Смерти презентацию делала?
18 514403
>>14373
Краткое.
Она потом из говен вокруг собирает обратный заряд и нейтральной становится.
19 514409
>>14328

>Зачем тратить деньги на космонавтов, решать вопросы со снаряжением, припасами, запасами кислорода?



Это многогранный вопрос. Люди начали летать в прошлом веке, еще тогда они понимали, что на ракетах с химическим двигателем много не налетаешь, но все ждали новых двигателей или вообще даже новые физические принципы перемещение. Но ничего этого так и нет. Люди летают по инерции, типа раз Гагарин полетел, то и нам надо летать. А если сейчас перестать летать, то я даже не знаю что будет, может вообще будет психологическая травма у общества и на космос все забьют болт.

>Технологии уже сейчас позволяют собирать ресурсы на Луне и в поясе астероидов.



Это бессмысленно. На Земле есть все нужные ресурсы и добывать их тут гораздо дешевле, чем в космосе.

>Ядерные космические двигатели изобретены



Технически его реализовать не получается. В теории он работает, а вот на практике нет. Ядерные двигатели это похоже тупиковая ветвь.
20 514410
>>14351

>а разве тут можно решать с точки зрения выгоды и эффективности



Все дорогие проекты оцениваются с точки зрения экономики. Иначе нельзя.
21 514411
>>14373

>Как чёрная дура может сохранять заряд


Вероятно у черной дыры нет никакого заряда.
22 514431
>>14411
https://en.wikipedia.org/wiki/Charged_black_hole
В теории никто не мешает ей быть, на деле же вряд ли.
Я бы даже вспизднул что черная дыра гипотетический объект, но мы недавно, говорят, сфоткали одну.
23 514433
>>14431

>В теории никто не мешает ей быть, на деле же вряд ли.


Соглашусь.

>Я бы даже вспизднул что черная дыра гипотетический объект, но мы недавно, говорят, сфоткали одну.


Ну мы не "видели" ее горизонта, астрофизики говорят, что нужны чувствительные гравитационные телескопы и космический радиоинтерферометр с как можно большей базой, чтобы позыкать что там за "поверхность" и как ведут себя гравитационные волны, по их "поведению" можно понять что у нее с поверхностью и есть ли она.
24 514493
>>14000 (OP)
Сомневаюсь, что тут кто-нибудь знает тетеньку с оппика, несмотря на то, что

Cecilia was able to accurately relate the spectral classes of stars to their actual temperatures by applying the ionization theory developed by Indian physicist Meghnad Saha. She showed that the great variation in stellar absorption lines was due to differing amounts of ionization at different temperatures, not to different amounts of elements. She found that silicon, carbon, and other common metals seen in the Sun's spectrum were present in about the same relative amounts as on Earth, in agreement with the accepted belief of the time, which held that the stars had approximately the same elemental composition as the Earth. However, she found that helium and particularly hydrogen were vastly more abundant (for hydrogen, by a factor of about one million). Her thesis concluded that hydrogen was the overwhelming constituent of stars (see Metallicity), making it the most abundant element in the Universe.

Ну и на то, что она была first woman to be promoted to full professor from within the faculty at Harvard's Faculty of Arts and Sciences
25 514508
>>14308
человек наверное не выдержит такое долгое пребывание в условиях невесомости. Обратно уже почти точно не доберётся и скончается довольно скоро после прибытия на Марсе - если вообще сможет оказаться на поверхности
Ещё технические проблемы с приземлением(примарсение?) на марсианскую поверхность, из-за очень низкой плотности атмосферы. Пилотируемый корабль как марсоход приземлить на Марс нельзя
26 514522
>>14508

>Пилотируемый корабль как марсоход приземлить на Марс нельзя


Почему?
27 514547
>>14410

>иначе нельзя


Можно.
28 514570
>>14403
Ну а если говна вокруг почти не будет, то долго будет заряженной - не в этом суть. Почему поле сразу не исчезает, хотя заряд уже провалился? Вон говорят исчезноевение инфы в чд, а заряд к этой инфе схуяли не относится?
29 514579
>>14570
Обожди, тут вопрос поважнее.
А с хуев ли это проблема какая-то, это ваше исчезновение инфы? Ну исчезла, ну и хуй с ней, какая разница?
Вон она испарится через квадриллион лет кварками рандомными, в чем проблема?
30 514584
>>14579
Потому что тогда её строение и процессы невозможно описать в рамках квантмеха, ото же даёт безосновательную экстраполяцию, что вся масса - в бесконечно плотной точке/кольце. Это никак ни смоделировать, ни проверить, вообще никаких подходов. Т.е. чд не поддаётся научному подходу, не вписывается в научную картину мира, неконструктивная смесь математической абстрактности и суеверных домыслов. Любая внятная теория будет предпочтительней.
31 514599
>>14584
Все равно непонятно в чем проблема.
Вон у нас БВ случился из какой-то математической абстракции маленькой точки, вселенная заспавнитась таким образом.
Как заспавнилась так и распидорасится черной дырой.
Масса сохраняется. А масса это энергия. Какая вам еще информация нужна?
32 514640
Как солнечный спектр может иметь линии поглощения металлов, если они все утонули в ядро, а фотоны из ядра миллиарды триллионов раз поглощаются-излучаются, прежде чем в космос улететь?
33 514645
>>14599

>Все равно непонятно в чем проблема.


Проблема в СИНГУЛЯРНОСТИ. Эту ахинею астрофизикам подкинули математики и астрофизики на нее клюнули. Когда математик не может свои долбанные уравнения решить, то он все валит на сингулярность, дескать тут ничего решить нельзя, возникает ебаная сингулярность.
34 514646
>>14599

>Вон у нас БВ случился из какой-то математической абстракции маленькой точки


БВ это гипотеза. Есть еще гипотеза столкновения "бран" (многомерных хуиток в теории струн). Дескать одна брана "упала" на другую и моментально везде случился БВ и никакой тебе маленькой точки.
35 514647
>>14640

>Как солнечный спектр может иметь линии поглощения металлов, если они все утонули в ядро,


С чего бы они утонули? Они же не жидкие, это мать его плазма!
36 514649
>>14647
Потому что тяжелее, гравитация, все дела, и времени дохуя на это было. Или, может, частично тонут, но как тогда смогли точно определять состав звезды в целом? Если на солнце аккуратно положить астероид (ну чтоб он не погрузился в него под действием импульса), его вещество так и будет болтаться у поверхности миллионы столетий?
37 514654
>>14649

>Потому что тяжелее, гравитация


Компенсируется давлением излучения. Когда излучение станет слабее и гравитация победит, то Солнцу придет каюк.

>Если на солнце аккуратно положить астероид (ну чтоб он не погрузился в него под действием импульса), его вещество так и будет болтаться у поверхности миллионы столетий?



На поверхности Солнца наблюдается значительная конвекция, там все попердывает и постоянно перемешивается поэтому эта часть Солнца называется конвективной зоной.
38 514669
>>14348

>Почему не навали кучу земли скажем в 20-40-60 и т.д. км


Во-первых, это ебически дорого. Во-вторых, твоя куча земли просто рассыплется под своим весом. Предел прочности гранитов под собственным весом - что-то около десятка километров вроде, как раз чуть больше самых высоких гор. Дальше неизбежно начнется разрушение. Да и насыпать гранит как-то сложно, максимум щебень (ну ты понел) или бетон, но бетону нужно много времени на застывание. Ну и в-третьих, как уже сказали, это не такой уж гигантский выигрыш для вывода на орбиту.
39 514696
>>14654

> Компенсируется давлением излучения. Когда излучение станет слабее и гравитация победит, то Солнцу придет каюк.


Это тут причём? Земле давление тоже не даёт в чёрную дуру превратиться, что не помешало почти всем тяжёлым элементам в ядро утонуть. Почему в солнце так же не тонут? Или тонут, но каким образом излучают тогда?
40 514701
>>14696

>не помешало почти всем тяжёлым элементам в ядро утонуть.


Но ведь у них там ядерная реакция начнется?
41 514702
>>14696

>Это тут причём?


При том, чо в центре Солнца идут ядерные реакции которые производят много излучения поток которого так пиздато греет меня сейчас.

>Земле давление тоже не даёт в чёрную дуру превратиться



Это ДРУГОЕ. На Солнце не давление вещества не дает ему схлопнуться, а давление излучения, потока фотончиков из центра Солнца. А почему металлы не утонули так потому что масса Солнца неебическая и слой над ядром окружен слоем неебичеки сжатого давлением водорода и гелия, там все настолько плотно сжато как, что ты даже представить себе не можешь, настолько сжато что никакого перемешивания вещества не происходит. Вот скажи, ты видел когда-нибудь водородную плазму под давлением имеющую плотность вольфрама? Вот и какие тебе металлы будут тонуть в такой плазме?
42 514710
>>14349

>Космос - это не как высоко, а как быстро, большая часть энергии ракеты уходит на разгон вбок.



значит нужно разгонять ракеты на рельсах и запускать как бы горизонтально?
43 514713
>>14710
Надо взрывать паровозы.
image.png32 Кб, 1875x862
44 514718
>>14710
Собственно, вот иллюстрация. В чем проблема такое заебошить?
45 514726
>>14718
ракета размером с великобританию судя по масштабу?
46 514728
>>14702
Ну конвективная зона мешает же вещество. А в больших звёздах и ядра вроде конвективные. И несмотря на то, что всё вещество перемешивается, для переноса вверх тяжелых ядер будет нужно больше энергии и они, в среднем, будут ниже концентрироваться... по логике.
47 514737
Что почитать про химию космического пространства? Конкретно про пыль, газ, поверхности астероидов, комет итд. Там вроде много чего интересного происходит, даже всякая органика образуется. Можно на английском.
48 514748
>>14726
если бы я нарисовал в масштабе, то чтоб увидеть ракету, тебе бы пришлось доставать микроскоп, который ты используешь когда хочешь увидеть свой член
15452533255c1ab1cd97db72.16779224.jpg64 Кб, 666x1000
стоит ли дарить парню-физику подвеску из настоящего метеорита? 49 514758
у моего молодого человека скоро день рождения. хочу ему приподнести такой необычный подарок, но я немного боюсь что он обидется т.к подумает что метеорит это в целом простая железяка и мол только деньги зря потратила. скажите пожалуйста, имеют ли метеориты какую-либо ценность, уникальность в плане состава? спасибо.
50 514760
>>14758
Подари ему хороший отсос и приведи подругу
51 514761
>>14154
Чому горячо то?
52 514762
>>14760
А подругу зачем? Подруга - куколд?
54 514801
>>14726
>>14748
тогда нахрена ты землю вниз загнул, идиот?
55 514805
Может ли звезда ебнуть без остатка ? Чтобы одно облако газа осталось
56 514806
>>14805
Да.
57 514810
>>14758
В метеорите могут содержаться споры инопланетных микроорганизмов. Это опасно.
58 514813
>>14801
ты пукнутый или что?
59 514816
>>14718

> В чем проблема


Да как же вы заебали. "В чем проблема построить амбар высотой 50 км, мы же можем построить 50-метровый амбар, давайте просто увеличим его в 1000 раз?" В квадрате-кубе основная проблема, а так же в том, что это только по объему работ миллиард амбаров, блядь.

Ты не можешь просто пропорционально увеличить что-то в N раз и ожидать, что оно по-прежнему будет работать. Как минимум давление на основание вырастет в N раз (масса в кубе, площадь основания - в квадрате). А значит 1) основание разрушится, ну разве что ты его из цельных нанотрубок сделаешь; 2) вся эта хуйня продавит земную кору и утонет в мантии. А еще, ты ожидаешь, что человечество резко начнет производить кубокилометры стройматериалов для твоего очередного мегапроекта? На какие шиши? Из чего? Какими мощностями? Лень считать, но подозреваю, что если собрать все постройки человечества за всю историю, они окажутся меньше этой ебалы.
60 514818
61 514819
>>14816

>Ты не можешь просто пропорционально увеличить что-то в N раз и ожидать, что оно по-прежнему будет работать



Ты тоже пукнутый. Что мешает построить типа эстакаду высотой 50 метров и длинной 25. Это как мост только легче потому что не в воде. Такие параметры будут у хуйни которая идет ровно горизонтально и не загибается. А можно и выше/длиннее
62 514821
>>14819
Ничего не мешает. Кроме того, что в этом нет смысла, потому что твоя ракета сразу же разуплотнится, когда попробует набрать 8 км/c в атмосфере околоземной плотности. Твоя хуйня на уровне Земли никакого смысла не имеет. Да и строго горизонтальной ей быть не надо. А там, где смысл есть - как минимум километров 10, на глазок (может и 30), - она станет тех самых циклопических и совершенно недостижимых размеров, которые угадываются по твоей картинке первой картинке.
image.png401 Кб, 1045x1634
63 514835
>>14737
Гуглится на раз, но как-то непонятно что тебе конкретно охота, тема обширная же.
Ну эта, если именно кометы 9783642748073
А так - водород да гелиум.
Потом звезды ХУЯК - и вся таблица менделеева.
А потом молекулы воды, аммиака и прочих простых соединений и все заверте...
image.png31 Кб, 350x131
64 514836
>>14805
Нет.
СОВСЕ МАЛАЯ не ебнет, а потухнет.
МАЛАЯ ебнет в белого карлана.
БОЛША ебнет в нейтрониевую звезду.
а ОСНЕ БОЛЬШАЯ в чорную дырень.

>>14806
С чего ты взял-то?
65 514837
>>14821
Че вы сразу ему так не ответили-то?

>>14819
Такая хуита имеет смысл на безатмосферных телах типа луны. Там, собственно, куча фантазеров и фантастов разных мастей и предлагают строить ЭМ-ускоритель, т.к. нет никаких проблем выйти на орбиту на высоте 10 метров (главное потом пропукайся еще повыше, чтоб не ебнуться назад)
Проверить можешь в огурцаче просто разогнавшись быстро на колесиках по поверхности моря Минмуса, например. Очень наглядно. Альцо поймешь чому такая ебала на земляшке не канает.
66 514839
>>14837
Да там много проблем. Например, для достижения 8 км/c даже при 10 g нужно 320 км, а вовсе не 25 (опять циклоп). Или технические проблемы опоры на рампу (иначе смысл теряется) на такой скорости: трение, например. Да и вообще, все потери на сопротивление гравитации в районе 10-20% вроде, бОльшая часть дельты-в именно для разгона нужна. И тут рампа никак не поможет.
67 514842
>>14737

>Что почитать про химию космического пространства?


Для начала послухай и реши надо ли тебе оно. А то вдруг ты ничего не поймешь.

https://www.youtube.com/watch?v=yU-MnmLdeGg
68 514843
Не следуй совету этого анона >>14842
Русские видео не смотри ни в коем случае если английский знаешь. Только если не знаешь - смотри переводы.
69 514845
>>14843

>Русские видео не смотри ни в коем случае


Чому?
70 514846
>>14845
Почему-то слишком легко нарваться на антинаучную или просто бестолковую парашу. Сразу деза уровня РенТВ от уверенно звучащего пиздобола в уши польется.
Про химию каналов, увы, не знаю, не интересовался про космологию есть PBS Space Time, крайне рекомендую.
71 514849
>>14846

>Почему-то слишком легко нарваться на антинаучную или просто бестолковую парашу.


Согласен. Но Дмитрий Зигфридович Вибе доктор физико-математических наук и заведующий отделом физики и эволюции звёзд Института астрономии РАН. Так что не ссы, смотри смело.
73 514878
>>14821

>когда попробует набрать 8 км/c


Кто тебе сказал что нуно набрать именно такую скорость? Мы не выстреливаем ракетой а даем ей некий стартовый разгон, допустим 3-4 км/с, чтоб меньше топлива тратить.
74 514879
>>14878
И она о плотную атмосферу останавливается нахуй, умник.
75 514881
А у России есть радиотелескопы миллиметрового диапазона?
image.png94 Кб, 987x454
76 514882
>>14881

>радиотелескопы миллиметрового диапазона


https://ru.wikipedia.org/wiki/Радиотелескоп_МГТУ_имени_Н._Э._Баумана
77 514886
>>14882
Раз он есть, почему нет ни одной статьи о нем в Вики на других языках?
image.png262 Кб, 1433x1206
78 514889
>>14886
https://en.wikipedia.org/wiki/List_of_radio_telescopes
Потому, что никто не запилил про него статью.
Ты можешь помочь проекту написав эту статью.
79 514892
>>14889

>Потому, что никто не запилил про него статью.


Почему раз он такой крутой? Про все крутые радиотелескопы статьи есть, а про этот почему-то нет?
80 514897
>>14879
Нет, потом она дальше на пердаковой тяге. просто будет стартовать не как сейчас с 0 км/с а с 3 км/с
81 514966
>>14839
Слыш, 8км/с это, внезапно, на 1/3 больше скорости звука в сталисели.
Из чего этот школ будет делать колёсья, рельсы, валы, подшипники и всё вот это вот?
Ды даже при лимите 3км/с в механизмах будут элементы с большей скоростью взаимодействия.
82 514967
>>14892
Ответ уже есть в посте: потому, что никто не запилил. Про много существующих телескопов нет отдельной статьи.
83 514969
>>14897
Ты понимаешь, что ты горишь на 10 махах летя близко к земле и тебе щас надо разворачиваться наверх иначе сгоришь нахуй. Или не понимаешь? Или кажется, что ты можешь так полчаса полететь сжигая топляк против торможения атмосферы чтоб наконец по горизонтали выбраться к менее плотным слоям?

Запусти огурцов, ей-богу, там наглядно понятно будет что к чему. Только поправку не забудь на то, что кербин - 1/10 земли и корабли чуть более волшебные чем ИРЛ.

Если доходит что атмосфера дохуя мешает и ты захочешь таки наверх подняться на малом газу и там потихоньку выбираться, то ты только что схему взлета Скайлона.
84 514973
>>14969

>Ты понимаешь, что ты горишь на 10 махах летя близко к земле


Можно лететь и абляцировать. Например.
image.png595 Кб, 1184x492
85 514975
>>14973
Зачем, во имя чего?
86 514976
>>14973
Ты так и не понимаешь сути, тебя весь путь тормозит атмосфера, даже если ты не сгораешь, ты кучу топлива впустую сжигаешь на поддержание скорости.
87 514977
>>14975
Ради лулзов, например.
88 514979
>>14976

> ты кучу топлива впустую сжигаешь на поддержание скорости.


А если мое топливо это атмосфэра?
89 514980
>>14977
Вот так и получаются НЛО. Никто не поверит что какой-то уебан сжег 8000 тонн керосина чтобы прочертить яркую прямую абляции и выхлопа над европейской частью страны.

>>14979
А я поэтому писал топливо. Если ты даже ГПВРД запряжешь и сможешь обуздать воздух и сжигать только топливо... ты все равно будешь сжигать дохуя топлива.

Сейчас слово за слово, торг за торг и до Скайлона договоримся, базарю.
image.png477 Кб, 700x546
90 514982
>>14980

>до Скайлона договоримся


Ну это хуже всего
91 514986
>>14980
Ты не понял. Топливо это атомы азота и кислорода, которые путем нехитрых ядерных превращений трансмутируем в другие элементики и маем немного энергии.
92 515000
>>14982
Так неча было на трамплинах скакать, взлетал бы как белый человек на вертикальной ракете или хотя бы по петле Лофстрома карабкался, другой разговор был бы.

>>14986
Так что ж ты сразу не сказал. Заодно энергетический кризис решил, вот умница!
93 515008
>>15000

>Заодно энергетический кризис решил, вот умница!


И атмосфэрку вам немного подобновил.
image.png85 Кб, 1100x620
94 515010
>>15008
Это мы и сами умеем.
17tLVHy-fhU2yHBw1Xgp1jw.jpeg324 Кб, 1350x901
95 515289
Почему вселенная молчит?
sage 96 515291
>>15289
потому же, почему
97 515297
>>15289
Что понимается под "молчит"?
Она вовсю пиздит на всех длинах волн спектра.
98 515298
>>15297
Имеется в виду наличие других разумных видов.
99 515299
>>15298

>других разумных


>других


>разумных


Какая смешная обезьяна.
100 515301
>>15298
Потому, что не нашли пока нигде.
Ну разве что дельфины и несколько штук приматов на этой планете есть из разумных.
101 515302
Лингвистика космоса вопросы. Я думаю, они изысканно тупы.
Почему Меркурианский, Венерианский, Марсианский, итд, а не Венерский, Марсский, Юпитерский етц?
Как будет такое же прилагательное про Землю? Земельный? Земляной? Землианский?
Почему юпитерская орбита будет Jovian а не jupiterian?
Почему у нас не так?
Почему при этом marsian, а не aresian?

Я еще один вопрос хотел, но забыл.
102 515303
>>15301

>на этой планете


Разумной жизни нет.
103 515306
>>15303
Подожди, а как же Нил Тайсон, Митио Каку, Брайан Кокс, Дольф Лунгрен, Григорий Перельман, Барак Обама, Рик Роснер, Рик Санчез, Майкл Висос, Дерек Веритасиум, Твайлайт Спаркл, Хайден Кристенсен и Стивен Хокинг?
104 515308
>>15302

>Как будет такое же прилагательное про Землю?



Грязный. Настоящая название нашей планеты - Грязь.
105 515311
>>15306
Благодарю за иллюстрацию.
image.png220 Кб, 1024x768
106 515319
>>15308
Не, грязный это обмазанный грязью, а не принадлежащий/относящийся к грязи.
Тогда грязевой уж.
Апоапсис грязевой орбиты МКС составляет 400 километров.
Во, так заебись.
Грязная орбита ж не звучит. Как будто в огурцаче глюк с орбитой словил.
Noice..mp4620 Кб, mp4,
1280x720, 0:03
107 515322
>>15311
Не за что. Я рад, что смог убедить тебя в наличии разумной жизни простым примером.
108 515324
>>15322

>этот днотадаунский говноролик


Игроки в жопа-2 - исчерпывающее и окончательное доказательство.
Разумной жизни на этой планете нет.
109 515327
>>15324
Обожди, то что днотапитухи используют что-то не значит что это их, этот мемос пошёл с форчка и YTP.
YTP - да, доказательство имбецильной жизни на этой планете.
110 515329
>>15327
Значит.
Зашквар невозможно провернуть назад.
111 515371
>>15324
Наличие одноклеточных - доказательство отсутствия многоклеточных?
112 515406
>>15371
Да.
113 515458
>>15406
>>15324

>Да


Твой ответ — исчерпывающее и окончательное доказательство.
Отрицательных ответов на этой планете нет.
114 515507
>>14154

>около 1500 градусов


Каких градусов?
115 515545
>>15507
Градусов Делиля.
116 515546
>>15302
Бамп. Почему за день никто не помог?
117 515554
>>15545
иди нахуй
118 515557
С какой высоты нужно упасть на Титане, чтобы разбиться нахуй?
119 515559
>>15289
Пруфы есть что молчит? Ты сам проверял? Может там во всю ящерики с птаагами срутся на всех частотах.
120 515561
Допустим, у нас есть кусок говна с консистенцией творога с массой Солнца, какие процессы в нем будут происходить и в какой последовательности? Как я понял, он начнет переплавливаться во что-то другое в центре и постепенно затронет все остальные слои? Через какое время говно полностью переработается?

Образование у меня 4 класса церковно-приходской, объясните с пояснениями пожалуйста.
121 515562
>>15546
Потому что соси хуй, быдло
122 515564
>>15557
Высота наверное обратно пропорциональна ускорению свободного падения. Вот у нас уже с трех метров больно падать, значит на Титане это будет 3 * 9.8 / 1.352 = 21.74556213017752 метров.
123 515566
3 факта почему ты думаешь, что есть внеземная жизнь/ее нет
124 515567
>>15561
Оно немного сожмется и внутри ебанет от поднявшейся температуры. Творог постепенно переработается в тяжелые элементы и станет ядром. Вся куча с течением времени будет грется, пока не раскалится и не будет светить.
125 515569
>>15567
То есть можно создать говяное солнце и освещать планеты говяным светом? Ничего себе!
126 515571
>>15566
1. Эволюция хуйня и не работает.
2. Предыдущего факта достаточно.
127 515573
>>15569
Да. Пофиг из чего оно будет сделано, оно все равно будет светить.
128 515587
>>15566
Скорее всего жизнь есть и есть рядом т.е. в пределах нескольких сотен парсеков. Но то что она разумна кажется маловероятным.
129 515600
>>15571
Слишком жирно.
130 515601
>>15573
Не, не пофиг. Из железа не будет.
131 515607
>>15564
Так там же еще плотная атмосфера.
132 515615
>>15607
Ты думаешь, что там терминалка медленнее скорости при которой человек ломает ноги и он будет падать как в киселе?
133 515618
Какие есть комплунктерные игори в которых можно лампово посычевать на другой планетке?
Возраст игры не важен абсолютно, можно аж с зарождения 16-цветного графена.
Огурцы и бесчеловечное небо не предлагать.
134 515619
С какого размера камень не прилетит целиком в планетку а будет распидорашен на подлете?
Насколько большая йоба может вот так мимо земляшки прилететь, цепануть атмосферу и улететь, чутка поджарившись? Может ли это быть кото клизменно?
135 515621
>>15618

>Какие есть комплунктерные игори в которых можно лампово посычевать на другой планетке?


Doom 2, Doom 3, Doom Eternal (уже скоро).
image.png436 Кб, 838x557
136 515622
>>15561
Самое ближайшее есть наглядно для тебя - это планетоид из хомячков кротов: https://what-if.xkcd.com/4/
На русской мове: https://chtoes.li/a-mole-of-moles/
Подробно смачно описано как их распидорасит и будет каша из фарша покрытая мягкой меховой оболочкой.
4.jpg28 Кб, 400x325
137 515623
>>15619

>Может ли это быть кото клизменно?

138 515624
>>15621

>лампово посычевать


>Doom


Джон Сталверн ждал. Светильники над ним мигнули и искрнули в воздух. На базе были демоны. Он не видел их, но ожидал их годами. Его предупреждения Палковнеку Джосону не были услышаны, а сейчас уже слишком поздно. Давно уже слишком поздно, в любом случае.

Джон был космодесантником четырнадцать лет. Когда он был молодым, он смотрел на космолеты, и сказал папе "Я хочу быть на кораблях, папочка."

Папа сказал "Нет! Ты будешь УБИВАННЫЙ ДЕМОНАМИ"

Было время, когда он ему верил. Затем, года он постаршенел, он перестал. Но сейчас, на космической станции базы ОАК он знал что там были демоны. "Это Джосон" протрещало радио. "Ты должен драться с демонами!"

Так что Джон хватал свою палзменную винтовку и взорвал стену.

"ОН ХОЧЕТЬ УБИТЬ НАС" сказали демоны

"Я выстрелю в него" сказал кибердемон и он выстрелил ракетными снарядами. Джон плазмнул в него и попытался подорвал его. Но потом потолок упал и они были в ловушке и не могли убивать.

"Нет! Я должен убивать демонов" закричал он

Радио сказало "Нет, Джон. Ты и есть демоны"

И тогда Джон был зомби.
139 515625
>>15289
Потому, что звук не распространяется в вакууме.
140 515626
>>15624
Финал катастрофически прекрасен.
141 515627
>>15626
Недаром это один из почетнейших текстов в коричневом фонде копипаст про игры.
142 515628
>>15625
Чой-то? В космосе не везде вакуум, есть огромные газопылевые облака которые наполнены звуками, звуковыми ударными волнами которые создают неоднородности структуру облака и формируют звезды. В НАЧАЛЕ БЫЛО СЛОВО.
143 515629
>>15618
Ancestors: The Humankind Odyssey. Я вообще на нем сижу как на героине.
image.png24 Кб, 500x47
144 515630
>>15629

>Я вообще на нем сижу как на героине


>Planned Release Date: 2020


Но как? Ты из будущего капчуешь?
145 515631
>>15628

>В НАЧАЛЕ БЫЛО СЛОВО.


И слово было РЯЯЯЯ!
146 515632
>>15630

>Но как?


Он уже выпущен.

https://en.wikipedia.org/wiki/Ancestors:_The_Humankind_Odyssey

Release
August 27, 2019
147 515633
>>15632
А в стиме чета нет.
148 515634
>>15633

>А в стиме чета нет.


Чойта?
149 515638
>>15634
А, бля, там кароч Эпик им заплатил чтобы те издавались у них эксклюзивно.
150 515639
>>15638
Так игруля просто шикарная, сейчас все на ней сидят. Брат вроде жив, тоже гамает, зависимость есть.
Стикер191 Кб, 500x500
151 515760
>>15302

>Как будет такое же прилагательное про Землю? Земельный? Земляной? Землианский?


Земной
152 515763
Я тут делаю "Инновационный" газовый котел для воздушного отопления с аж двумя теплообменниками, долго рассказывать.

В общем, там установка био-метановая с очисткой метана до уверенно выше 90%

Установка за сутки выдает 3 куба чистого метана, там будет принудиловка...

Мне нужно сжигать метан с максимальным КПД без давления, сколько кубов воздуха смешивать на 3 куба метана,

И как лучше смешивать, там будет условно воздуховод в 1.5 метра, там можно производить смешение а дальше горелка, или лучше гарелку чисто под метан а воздух загонять сбоку?

Вроде как в ракетных двигателях максимально замешивают в смесителе...
153 515766
>>15763

>гарелка



Горелка

комфорка
154 515773
>>15763

>Вроде как в ракетных двигателях максимально замешивают в смесителе...


Вейт, а разве в ракетных двигателях его вообще замешивают перед сжиганием? Оно разве не при первом же контакте с окислителем самовоспламеняется?
155 515798
>>15763
Если смешать а потом поджечь - то будет взрыв, как в ДВС и вакуумных бомбах.
Если нужно горение с высоким КПД - то воздух направлять на пламя, как в ракетных печах.
156 515813
>>15798

>то будет взрыв



А детонационную печь можно замутить?
Изображение-663.jpg149 Кб, 800x494
157 515815
Если марсоебам так важно сохранение жизни "Планета в порядке - это людям пиздец", почему при каждом запуске не цепляется какая-нибудь килограммовая капсула с десятикилограммовым движком, с самыми живучими организмами и не отправляется нахуй из солнечной системы. Понятно что на запуске на Марс бабло гораздо лучше пилится. Но именно мой план работоспособный или все же с орбиты не так просто улететь?

Селедка с подвеской, напомнила мне про передачу, которую я смотрел про метеоритоискателей. Тупо гугланул металлоискатели, и ахуел, что низ начинается чуть ли не со ста баксов. Стоит ли пытаться вкатиться в искателя, не для наживы ради, а просто как хобби. В радиусе 30 км от моей деревни примерно такой рельеф как на пике, то ли ледниками так по пидорасило, то ли хз еще от чего.
Такой рельеф скорее плюс или минус к поискам? По ощущениям вроде как более легкие и мелкие частицы должны слетать с этих горочек и на верхушках много чего интересного должно быть или все же равнины лучше в этих делах?
158 515817
>>15815
Шанс попасть такими запусками в какую нибудь звездную систему - 0%.
Можно пролететь всю галактику насквозь в каком нибудь space engine, и ты никогда не встретишься со звездой.
159 515818
>>15601
Будет. Солнце греют не ядерные реакции, а само вещество. Атомы вещества ведь диссипируют в процессе своей жизнедеятельности, образуя вокруг себя поля, которые мы называем всякими научными терминами. Флуктуации этих полей колеблют соседние атомы - получается тепло. Один атом выделяет мало тепла, но если атомов много, то тепло накапливается.
160 515819
>>15618
PioneerSpaceSim
161 515821
>>15818
Нихуя се, атомы щекотят друг друга и тепло появляется. Я думал все проще, атомы разрушаются - выделяется огромное количество энергии, тоже самое и при соединении водорода в гелий, а тут во как.
162 515861
>>15773

>Оно разве не при первом же контакте с окислителем самовоспламеняется?


это справделиво только для гиперголических пар
ндмг + ат
остальное надо поджигать
163 515862
>>15815
только один раз
164 515878
>>15815
жизни людей блядь
бактерии и так выживут пока солнце не поглотит землю
165 515916
>>15798

>Если смешать а потом поджечь - то будет взрыв, как в ДВС и вакуумных бомбах.



Надо мутить котел на микро взрывах! вроде как энергетика там на порядок выше!
166 515926
>>15821
Ядерные реакции бурно происходют при формировании Солнца, а когда оно сформировано, то ядро представляет собой кучу тяжелых элементов, которым уже некуда дальше объединяться и ядерные реакции происходят по минимуму.

Считаем сколько примерно мощности выделяет один протон. Мощность Солнца делим на массу Солнца и множим на массу протона.
3.828e26 / 1.9885e30 * 1.673e-27 = 3.2206406839326e-31 Вт или 2.0101490359649e-12 эВ/с
Как видим, величина достаточно мизерная, то есть протон вполне способен столько выделять своими флуктуациями.
167 515938
>>15926
Выделил тебе за щеку.
168 516036
>>14000 (OP)

Что там за макромолекулы нашли на Энцеладе?Это чё??Вроде там и вода есть
169 516040
>>15306

Алекса Филипенко забыл
170 516090
>>16036

>Что там за макромолекулы нашли на Энцеладе



Органические соединения. Как простые типа метана, этана, ацетилен так и более сложные - метанол, этанол и пр.
171 516116
>>14042
унылое зрелище
Невозможно сосредоточится
У хохлов там ничего не менялось с 1980х
Такой монотонный бубнёж даже для одарённых ботаников едва на грани приемлемости

Для обычных и тупых людей необходима компьютерная графика - мультипликация и приятная дикция
172 516118
Аноны, читаю историю советской лунной програмы, нашел пик. Вопрос: что за круглая херня на конце поздних версий Н-1 ?! Это такой обтекатель?! нихуя не пойму.
173 516119
>>16116

>Невозможно сосредоточится


У тебя СДВГ?
174 516121
>>16118
Православный купол.
175 516127
>>16121
жирно
image.png113 Кб, 3136x159
176 516143
>>15926

>Считаем сколько примерно мощности выделяет один протон


Чебля

>Мощность Солнца делим на массу Солнца и множим на массу протона.


ЧЕБЛЯДЬ
У тебя ВСЁ СОЛНЦЕ что ли участвует в реакциях, ебанат?
Не пори дебильную математику, за тебя все давно посчитано.
177 516144
>>16119
У меня НДМГ, но да, вместо бубнежа лучше книгу почитать. А если делаете познавательный материал, так хоть картинки рисуйте.

видео не смотрел и не собираюсь
Баба страшная(( 178 516162
Поставьте негра хоть на аву, да хоть кого-нибудь, умоляю!
Сил нет на страхолюдину смотреть
179 516163
>>16118
Как обычно в рунете информации сильно меньше, а астронавтикс нихуя не работает.
Это скорей всего ДЛБ, Долговременная Лунная База, но инфа 80%.
Алсо, на пикче у тебя Н-1М, у которой криогеника сверху используется.
180 516164
>>16162
НСФВ нажми и все.
181 516167
>>16164
это какая-то куклоприблуда? Помню была комбинация клавиш, чтобы заблюрить пикчи, напомни что-ли. хотя она вроде всё скрывает
182 516168
>>15815
Без геологического навыка, ты не отличишь метеорит от горной породы, лол
183 516170
>>16168
Всмысле не отличу? 80% того что долетает или металлические или с нихуевым содержанием металла. Любой черный (оплавленный/обожженный) камень это метеорит.
184 516172
>>16170
откровенная чернота смоется после пары дождей, покроется мхом, грязью. Есть черные базальты и в природе например, или кто-то в костре камень подержит и ты такой "Метеорит!"
185 516174
>>16172
https://www.youtube.com/watch?v=EZg4059i1fE
Первый видос по запросу "поиск метеоритов с металлоискателем"
1.20 проверка магнитом, открой любой видос по теме, там будет про магнит. Ну если, конечно, в говне моченые не наебывают обычных людей.
186 516193
>>16163
Да она, меня именно интересовала довольно интересная форма головной части. Явно что то грузовое, САС нет. А ссылку на источник что, это может быть ДЛБ не кинешь?! хотелось поподробней почитать.
187 516207
>>16174
Так на видео он говорит о том, чтобы искать свежий метеорит под эпицентром взрыва, куда осколки падают. То есть это нечто с конкретной позицией, известным составом, недавнее. А просто в рандомном лесу без наводки ты хуй чо найдешь. Будешь там по 100 раз наклоняться на каждый гвоздь, гайку, крышку из под пива.
188 516216
>>14042
Охуенные лекции. Когда работал репетитором по физике, смотрел у него что забыл со школы.
189 516241
>>16118>>16163>>16193
Первая пикча - выдумка Вэйда.
На фотографии справа - конструктивно-подобная модель РКН Н1-Л3 в масштабе 1:10, слева - КМП блока В РН Н1 без хвостового отсека с комплексом Л3 без головного обтекателя в масштабе 1:5.
190 516247
>>16241
Благодарю анон.
191 516261
Как думаешь, Патомский кратер - космического происхождения? Или это что-то иное?
192 516276
>>16261
Нет конечно, он же просто кричит о своём вулканическом происхождени.
193 516277
>>16276
Так нет, не нашли следов магмы и трубки магматической.
Так как возраст кратера имеет принципиальное значение, в 2008 году В. И. Ворониным (СИФИБР СО РАН) был предпринят массовый отбор спилов наиболее высоковозрастных лиственниц в нескольких метрах от внешней осыпи кратера и на самом кратере. В результате дендрохронологического анализа обосновано заключение, что около 500 лет назад в процессе формирования насыпного конуса и активной подвижки грунта произошёл массовый вывал деревьев и появилось новое поколение лиственниц с возрастом 400—480 лет.
194 516278
>>16277
А это и не обязательно, ёба. Блядь, да он даже не похож на импактные кратеры, ну ты чё?

>Геолого-геохимические исследования в составе комплексных экспедиций 2006, 2008 и 2010 годах свидетельствуют об эндогенных причинах образования кратера, которые, очевидно, связаны с развитием глубинного магматического процесса.


>Установлено, что эруптивный материал в виде глыб песчаников и сланцев среди известняков, выведенный на современную поверхность, был интенсивно карбонатизирован. В результате, в терригенных породах образовалась минеральная ассоциация карбонатных минералов: кальцит, сидерит, анкерит, флюорит. Обнаружение в пределах кратера отдельных глыб терригенных пород с аномальными геохимическими характеристиками даёт возможность предполагать существование на глубине субвулканического магматического тела, определившего вещественные особенности пород Патомского кратера.


>Предполагаемая ранее метеоритная гипотеза происхождения Патомского кратера не подтверждается проведёнными комплексными исследованиями, его образование связано с эндогенными процессами, главную роль в которых играло поступление глубинного потока газовых и флюидных компонентов.

195 516279
>>16277
да исключается метеорит в любом случае. Хули он такой конус образовал? от метеорита была бы просто чаша, которую заполнила бы вода, деревья и было бы болотце
Может просто пожар был в тайге, вот тебе и новое поколение лиственниц.
196 516281
>>16279

>да исключается метеорит


Геофизики георадарами нашли какое-то металлическое тело под землей, что-то там есть.
dobycha-rudy.jpg467 Кб, 900x600
197 516284
>>16281
ШОК В ЗЕМНОЙ КОРЕ ОБНАРУЖЕН МЕТАЛЛ
Анон, просто пойди и погугли как выглядят ударные кратеры, и сразу поймёшь что пытаться натянуть метеорит на Патомский кратер очень глупая затея.
198 516285
>>16281
естественные минералы какенибудь,или типа металлический ксенолит в глотке застрял.
199 516287
>>16284

>ШОК В ЗЕМНОЙ КОРЕ ОБНАРУЖЕН МЕТАЛЛ



Не просто металл, а некий компактный металлический продолговатый предмет.
200 516289
>>16287
Про металлоносные пласты породы не слышал никогда?
201 516293
>>16289
Геофизики, в отличие от тебя не могут объяснить причины появления этого металлического тела.
202 516295
>>16293
Это хорошо, правда в отличие от тебя никто из них высосать метеорит из нихуя не пытается, потому что умеют отличать ударные кратеры от других.
Ты серьёзно думаешь, что столкнувшееся с Землёй на ебанистической скорости тело зарывается в землю и лежит там одним куском, насыпав над собой аккуратный конус породы, при этом ещё и магическим образом не оставлет следов ударного расплава?
203 516316
Как выглядит график радиус-время ядра в момент коллапса в нейтронную звезду? Давление вещества постепенно гасит импульс или происходит серия упругих вибраций или по-другому вообще?
204 516334
>>16316
Может зависит от массы звезды, температуры, от наличия ассиметричностей во время коллапса?
205 516337
>>14000 (OP)
Можно ли гравитационным манёвром у Юпитера направить спутник и увеличенной скоростью прямо в ад нахуй к Солнцу?
206 516338
>>16337
Да, почитай про аппарат Улисс, они почти именно так и сделали. Только не прямо в Солнце, но изменили наклонение и направили к полюсам.
https://ru.wikipedia.org/wiki/Улисс_(космический_аппарат)
image.png14 Кб, 705x91
207 516371
>>16167
Наверх посмотри.
208 516372
>>16241
Я тоже тебя благодарю, анон.

Заодно извиняюсь за дезу про ДЛБ.
209 516374
>>16337
Можно. Но это не очень рационально.
Во первых, аппарат придется делать под слишком большой диапазон температур и энергии, поскольку излучение Солнца у Юпитера в ~25 раз слабее, чем у Земли.
Во вторых, отправка к Юпитеру очень затратна. По топливу гораздо выгоднее сделать несколько маневров подряд у Венеры и Земли.
Да и по времени будет не сильно хуже, так как один пролет к Юпитеру будет длится гораздо дольше, чем даже несколько пролетов Венеры. Скажем. упомянутый этим >>16338 Улисс стартовал в октябре 1990, а над полюсом с максимальной широтой прошел в сентябре 1994, то есть потратил около 4 лет. Parker Solar Probe за те же 4 года совершит 5 гравиманевров у Венеры из семи запланированных.
210 516392
>>16371
Спасибо.
211 516409
>>16143
Так я не про ядерные реакции написал, а про кое-что другое, о чем ты помыслить не в состоянии.
212 516662
>>16409
И нахуя ты про кое-что другое пишешь, если энергия солнца от ядерных реакций, шизик?
213 516663
>>16374
Бредишь, как раз от дальних планет наиболее выгодно по топливу упасть на солнце. По времени - да, невыгодно.
214 516670
>>16663

>Бредишь, как раз от дальних планет наиболее выгодно по топливу упасть на солнце.


Нет. До Венеры нужно меньше топлива, а наматывать маневры у нее и Земли можно сколько угодно. У того же Паркера запланированно аж 7 маневров.
Нужно ~3.35 км/с сверх второй космической, что бы долететь до Юпитера. Что бы начать наматывать круги мимо Венеры достаточно ~0,65 км/с.
215 516714
>>16662

>энергия солнца от ядерных реакций


Это не точно, никто же в солнышко не нырял и не проверял что у нее внутри происходит. То есть это теория, а не 100% факт. Так что вполне позволительно сочинять другие гипотезы и теории.
216 516741
>>16714

>это теория, а не 100% факт


Готовь ебальник
217 516766
>>16714

>Это не точно


А от чего же тогда на Солнце берется энергия?
218 516852
>>16766
В XIX веке вообще думали, что там уголь горит.
219 516854
>>16852
Где нибудь можно почитать истории открытий и представлений о космосе? Концентрированно, а не по 20 статьям бегать.
220 516867
>>16854
Энциклопедия для детей по астрономии от издательства "Аванта+", 2013 год.
На слова "для детей" не гляди, писали серьёзные астрономы на отличном научно-популярном уровне, вполне сойдёт и для взрослых.
221 516879
>>16714

>То есть это теория, а не 100% факт


О, креационист на моём спейсаче.
222 517061
>>16854

>Где нибудь можно почитать истории открытий и представлений о космосе?


Полуркай на ютубе лекции об истории астрономии, например лекции Язева или Сурдина.
223 517074
Такой вопрос - что будет если рядом будут находиться две двойные черные дыры, излучающие гравитационные волны в фазе друг на друга? Будут ли волны складываться, создавая локальное расширение/сжатие пространства-времени или это все бред?
224 517084
>>17074

>создавая локальное расширение/сжатие пространства-времени


Так гравиволны это и есть расширение/сжатие пространства-времени. Представь это как волны на воде от двух камне. Они будут интерферировать.
225 517185
>>17074
Они больше не объединены.
226 517226
Какова была б температура на Венере, будь она на орбите Марса?
227 517230
>>16714
Точно-неточно, а ты ничего и отдалённо близкого по убедительности взамен не предложишь. И у тебя обывательское представление о слове "теория". По-твоему, это когда кто-то остроумно пёрнул в лужу и все ему поддакивают на веру. Между тем как ситуация прямо противоположна, и теория постоянно подвергается всесторонним проверкам в том или ином виде.
228 517233
>>17226
Из-за парникового эффекта такая же, как и щас. Ну, мож, градусов на 100...200 пониже - один хрен не выжить.
229 517257
>>17233
Но ведь и атмосфера сама послабее была бы (и мож намного). Многие газы скомбинировались бы и осели, например. Углекислый газ затвердел бы в полярных шапках, там.
food-1681977640.png407 Кб, 640x605
230 517307
Что случится со свежим яблоком если его оставить на поверхности Луны в середине дня? Сразу же и через два месяца?
231 517315
>>16714
Назови мне хоть один 100% факт.
232 517328
Если с НОО сбросить дерево с комом земли и густой кроной(например очень пушистую ель), то сможет ли его крона с ветками и иголками послужить чем-то вроде атмосферного тормоза, чтоб на землю пизданулась не угольная пыль, а ствол, пусть и обгорелый?
233 517352
>>17307
Оно высохнет.
234 517354
>>17328
Не сможет, а вот кора и древесина могут служить теплозащитой, тут от толщины ствола зависит.
235 517356
>>17315
ОП - хуй.
236 517487
>>17257
Хуй ты рассчитаешь, какая там атмосфера была бы. Если температура будет хотя бы как на Земле, то никакой углекислый газ никуда не выпадет. Тут-то он не твердеет. Да и на Марсе он только на полюсах твердый и тает сублимируется летом. Вообще, температура и атмосфера очень сильно взаимосвязаны и просчитать что и как развивалось бы, если бы Венера 4 миллиарда лет была на орбите Марса нереально при нынешних знаниях.
237 517528
Вот это дерьмо будет радиоактивным или это обычные видимые фотоны? Если я увижу такую сверхновую, то нужно ли прикрываясь портфелем бежать к метро, или можно наслаждаться видом и доставать телефон?
https://twitter.com/CaliaDomenico/status/1178748703638740993
238 517558
>>17528
Разумеется, там весь спектр. Но оно не страшнее Солнца, от которого тебя защищает атмосфера.
239 517588
>>17558
Если так, то хорошо. потому что у меня в городе нет метро.
240 517596
>>17528
Хуясе, а я сегодня даже на улице не был.
241 517640
>>17588
А тебе Бетельгейзе его и не проложит.
242 517653
>>17640
Ну и нахрен она нужна тогда?
243 517663
Чем ближе к Солнцу, тем время течёт быстрее, в сравнение с земным?
244 517667
>>17663
Если испытывать ускорение (т.е. не крутиться вокруг и не падать свободно), то да.
245 517679
НазваниеСтоимостьПНОрбита
Ravn$0,2M0,1тССО; 500 км
NEPTUNE NS$0,25M0,0064т90°; 310 км
Spyder Orbital$1M0,009 тНЗО
Vector-R$1.5M0,066 тНЗО
Vector-RE1$2M0,035 тCCО; 800 км
Vector-H~$3M0,16 тНЗО
VLM$3M0,16 тНЗО
Jielong-1 (Smart Dragon-1)$6M0,2 тССО; 500 км
Куайчжоу KZ-1$6M0,43 тССО; 500 км
Electron$6,95M0,1 тССО; 500 км
Landspace LS-1$8M0,4 тССО; 500 км
Куайчжоу KZ-1A$9M0,25 тССО; 500 км
INTREPID-1$9M0,376 тССО; 500 км
CZ-11$10M0,435 тССО; 500 км
Terran-1$10M1,25 тНЗО
LauncherOne$12M>0,3тCCО; 500 км
RS1$12M0,875тCCО; 500 км
Minotaur I$12,5M0,335тССО; 740 км
Shavit 1$15M0,35 т90°; 240X600 км
Firefly Alpha$15M0,63 тССО; 500 км
Athena I$17M0,36 тССО; 800 км
Днепр$20M0,8 тССО; 700 км
Рокот / Бриз-КМ (Федер.)$21M1,15 тССО; 700 км
Союз 2-1б (Федер.)$23M8,35 т51,8°; 200 км
Minotaur IV$23,6M1,075тССО; 700 км
Союз-2-1В/БВ Волга$25M1,4 тССО; 835 км
CZ-2C$25M1,4 тССО; 600 км
CZ-2D$25M1,3 тССО; 600 км
PSLV-CA$29,3M1,2 тССО; 630 км
PSLV-XL$31,5M1,75 тССО; 630 км
Minotaur-C (Taurus XL)$35M0,93 тССО; 700 км
Союз 2-1б (Главкосмос 2018 )$35M8,35 т51,8°; 200 км
Epsilon$38M0,45 тССО; 500 км
Рокот / Бриз-КМ (Eurockot)$39M-$45M1,15 тССО; 700 км
Vega (2015)$44M1,5 тССО; 700 км
Циклон-4М$45M3,35 тССО; 700 км
GSLV Mark 2$45M2,5 тГПО dV=1800 м/с
Союз 2-1б / Фрегат (Федер.)$47M4,6 тССО; 820 км
Союз 2-1б / Фрегат (Главкосмос 2018 )$48,5M4,6 тССО; 820 км
Союз 2-1а / Фрегат (Starsem)$50M4,2 тССО; 820 км
Днепр (Iridium 2G)$51,8M0,8 тССО; 700 км
Pegasus XL$56,3M0,225тССО; 700 км
Зенит 3SLБ / ДМ SLБ$60M4,5 тГПО dV=1800 м/с
CZ-3A$60M2,6 тГПО dV=1800 м/с
CZ-4B$60M2,3 тССО
CZ-4C$60M2,9 тССО
Falcon 9 v1.2 (2016)$62M5,3 тГПО dV=1800 м/с
Протон М / Бриз М (Федер.)<$65M3,7 тГCО
Протон М / ДМ-03 (Федер.)<$65M3,0 тГСО
Протон М / Бриз М (ILS)~$656,5 тГПО dV=1500 м/с
Falcon 9 v1.2 (Iridium Next, 2016)$67,9M5,3 т86.4°; 667 км
CZ-3B$70M5,1 тГПО dV=1800 м/с
CZ-3BE~$70M5,5 тГПО dV=1800 м/с
CZ-3C$70M3,8 тГПО dV=1800 м/с
Союз СТБ / Фрегат МТ$80M4,9 тССО; 820 км
Antares 230$80-85M6,7 т51,6°; 200 км
Зенит 3SL / ДМ SL$80-90M6,5 тГПО dV=1500 м/с
Ariane 62$85M5,0 тГПО dV=1500 м/с
Falcon 9 v1.1 (AMOS 6, 2015)$85M5,5 тГПО dV=1800 м/с
Vulcan-Centaur 504 (2020)$85M4,75 тГПО dV=1800 м/с
Falcon 9 v1.1 (NLS II)$87M4,85 тГПО dV=1800 м/с
Falcon Heavy (2016)$90Mдо 8 тГПО dV=1800 м/с
Falcon 9 v1.2 (USAF GPS-III, 2017)$96,5M3,68 т55°; 20200 км
Falcon 9 v1.1 (USAF 2015)$97M4,85 тГПО dV=1800 м/с
Falcon 9 v1.2 (Sentinel-6A, NLS-II, 2017)$97M1,4 т66°; 1336 км
H-IIA 202$98M4,0 тГПО dV=1800 м/с
H-IIA 204$98M6,0 тГПО dV=1800 м/с
Atlas V 401 (2017)$109M4,767тГПО dV=1800 м/с
Atlas V 411 (2017)$115M5,964тГПО dV=1800 м/с
Atlas V 501 (2017)$120M3,778тГПО dV=1800 м/с
Atlas V 421 (2017)$123M6,903тГПО dV=1800 м/с
Atlas V 431 (2017)$130M7,715тГПО dV=1800 м/с
Atlas V 511 (2017)$130M5,248тГПО dV=1800 м/с
Ariane 64$130M11,5 тГПО dV=1500 м/с
Atlas V 401 (JPSS-2, NLS-II, 2017)$132,4M2,93 тCCО; 824 км
Falcon Heavy (2012)$135M21,2 тГПО dV=1800 м/с
Atlas V 521 (2017)$135M6,477тГПО dV=1800 м/с
Delta II 7320-10С~$140M1,6 тССО; 833 км
Delta II 7920-10С~$140M3,0 тССО; 833 км
Atlas V 531 (2017)$140M7,448тГПО dV=1800 м/с
Atlas V 541 (2017)$145M8,287тГПО dV=1800 м/с
Atlas V 551 (2017)$153M8,899тГПО dV=1800 м/с
Atlas V 401 (Landsat-9, NLS-II, 2017)$153,8M2,623 т98,2°; 705 км
Delta IV Medium+(4,2) (2014)$164M6,39 тГПО dV=1800 м/с
Falcon Heavy (Air Force 2015)$165M21,2 тГПО dV=1800 м/с
H-II B$182M16,5 т30,4°; 300 км
Атлас V 551 (USAF STP-3, 2017)$191,1МПОГСО
Атлас V 541 (MSL)$195М4,75 т11,2 км/с
Атлас V 551 (New Horizons)$213М0,478т16,21 км/с
Delta IV M+ (5,2) (2014)~$225M5,49 тГПО dV=1800 м/с
Delta IV Medium+ (5,4) (2014)~$225M7,3 тГПО dV=1800 м/с
Ariane 5 ES$240M20,0 т51,6°; 300 км
Ariane 5 ECA$240M9,5 тГПО dV=1500 м/с
Атлас V 541 (Mars 2020)$243М4,75 т11,2 км/с
Vulcan-Centaur 564 (2020)$260M ГПО dV=1800 м/с
Delta IV Heavy (2015)$389M14,220тГПО dV=1800 м/с
245 517679
НазваниеСтоимостьПНОрбита
Ravn$0,2M0,1тССО; 500 км
NEPTUNE NS$0,25M0,0064т90°; 310 км
Spyder Orbital$1M0,009 тНЗО
Vector-R$1.5M0,066 тНЗО
Vector-RE1$2M0,035 тCCО; 800 км
Vector-H~$3M0,16 тНЗО
VLM$3M0,16 тНЗО
Jielong-1 (Smart Dragon-1)$6M0,2 тССО; 500 км
Куайчжоу KZ-1$6M0,43 тССО; 500 км
Electron$6,95M0,1 тССО; 500 км
Landspace LS-1$8M0,4 тССО; 500 км
Куайчжоу KZ-1A$9M0,25 тССО; 500 км
INTREPID-1$9M0,376 тССО; 500 км
CZ-11$10M0,435 тССО; 500 км
Terran-1$10M1,25 тНЗО
LauncherOne$12M>0,3тCCО; 500 км
RS1$12M0,875тCCО; 500 км
Minotaur I$12,5M0,335тССО; 740 км
Shavit 1$15M0,35 т90°; 240X600 км
Firefly Alpha$15M0,63 тССО; 500 км
Athena I$17M0,36 тССО; 800 км
Днепр$20M0,8 тССО; 700 км
Рокот / Бриз-КМ (Федер.)$21M1,15 тССО; 700 км
Союз 2-1б (Федер.)$23M8,35 т51,8°; 200 км
Minotaur IV$23,6M1,075тССО; 700 км
Союз-2-1В/БВ Волга$25M1,4 тССО; 835 км
CZ-2C$25M1,4 тССО; 600 км
CZ-2D$25M1,3 тССО; 600 км
PSLV-CA$29,3M1,2 тССО; 630 км
PSLV-XL$31,5M1,75 тССО; 630 км
Minotaur-C (Taurus XL)$35M0,93 тССО; 700 км
Союз 2-1б (Главкосмос 2018 )$35M8,35 т51,8°; 200 км
Epsilon$38M0,45 тССО; 500 км
Рокот / Бриз-КМ (Eurockot)$39M-$45M1,15 тССО; 700 км
Vega (2015)$44M1,5 тССО; 700 км
Циклон-4М$45M3,35 тССО; 700 км
GSLV Mark 2$45M2,5 тГПО dV=1800 м/с
Союз 2-1б / Фрегат (Федер.)$47M4,6 тССО; 820 км
Союз 2-1б / Фрегат (Главкосмос 2018 )$48,5M4,6 тССО; 820 км
Союз 2-1а / Фрегат (Starsem)$50M4,2 тССО; 820 км
Днепр (Iridium 2G)$51,8M0,8 тССО; 700 км
Pegasus XL$56,3M0,225тССО; 700 км
Зенит 3SLБ / ДМ SLБ$60M4,5 тГПО dV=1800 м/с
CZ-3A$60M2,6 тГПО dV=1800 м/с
CZ-4B$60M2,3 тССО
CZ-4C$60M2,9 тССО
Falcon 9 v1.2 (2016)$62M5,3 тГПО dV=1800 м/с
Протон М / Бриз М (Федер.)<$65M3,7 тГCО
Протон М / ДМ-03 (Федер.)<$65M3,0 тГСО
Протон М / Бриз М (ILS)~$656,5 тГПО dV=1500 м/с
Falcon 9 v1.2 (Iridium Next, 2016)$67,9M5,3 т86.4°; 667 км
CZ-3B$70M5,1 тГПО dV=1800 м/с
CZ-3BE~$70M5,5 тГПО dV=1800 м/с
CZ-3C$70M3,8 тГПО dV=1800 м/с
Союз СТБ / Фрегат МТ$80M4,9 тССО; 820 км
Antares 230$80-85M6,7 т51,6°; 200 км
Зенит 3SL / ДМ SL$80-90M6,5 тГПО dV=1500 м/с
Ariane 62$85M5,0 тГПО dV=1500 м/с
Falcon 9 v1.1 (AMOS 6, 2015)$85M5,5 тГПО dV=1800 м/с
Vulcan-Centaur 504 (2020)$85M4,75 тГПО dV=1800 м/с
Falcon 9 v1.1 (NLS II)$87M4,85 тГПО dV=1800 м/с
Falcon Heavy (2016)$90Mдо 8 тГПО dV=1800 м/с
Falcon 9 v1.2 (USAF GPS-III, 2017)$96,5M3,68 т55°; 20200 км
Falcon 9 v1.1 (USAF 2015)$97M4,85 тГПО dV=1800 м/с
Falcon 9 v1.2 (Sentinel-6A, NLS-II, 2017)$97M1,4 т66°; 1336 км
H-IIA 202$98M4,0 тГПО dV=1800 м/с
H-IIA 204$98M6,0 тГПО dV=1800 м/с
Atlas V 401 (2017)$109M4,767тГПО dV=1800 м/с
Atlas V 411 (2017)$115M5,964тГПО dV=1800 м/с
Atlas V 501 (2017)$120M3,778тГПО dV=1800 м/с
Atlas V 421 (2017)$123M6,903тГПО dV=1800 м/с
Atlas V 431 (2017)$130M7,715тГПО dV=1800 м/с
Atlas V 511 (2017)$130M5,248тГПО dV=1800 м/с
Ariane 64$130M11,5 тГПО dV=1500 м/с
Atlas V 401 (JPSS-2, NLS-II, 2017)$132,4M2,93 тCCО; 824 км
Falcon Heavy (2012)$135M21,2 тГПО dV=1800 м/с
Atlas V 521 (2017)$135M6,477тГПО dV=1800 м/с
Delta II 7320-10С~$140M1,6 тССО; 833 км
Delta II 7920-10С~$140M3,0 тССО; 833 км
Atlas V 531 (2017)$140M7,448тГПО dV=1800 м/с
Atlas V 541 (2017)$145M8,287тГПО dV=1800 м/с
Atlas V 551 (2017)$153M8,899тГПО dV=1800 м/с
Atlas V 401 (Landsat-9, NLS-II, 2017)$153,8M2,623 т98,2°; 705 км
Delta IV Medium+(4,2) (2014)$164M6,39 тГПО dV=1800 м/с
Falcon Heavy (Air Force 2015)$165M21,2 тГПО dV=1800 м/с
H-II B$182M16,5 т30,4°; 300 км
Атлас V 551 (USAF STP-3, 2017)$191,1МПОГСО
Атлас V 541 (MSL)$195М4,75 т11,2 км/с
Атлас V 551 (New Horizons)$213М0,478т16,21 км/с
Delta IV M+ (5,2) (2014)~$225M5,49 тГПО dV=1800 м/с
Delta IV Medium+ (5,4) (2014)~$225M7,3 тГПО dV=1800 м/с
Ariane 5 ES$240M20,0 т51,6°; 300 км
Ariane 5 ECA$240M9,5 тГПО dV=1500 м/с
Атлас V 541 (Mars 2020)$243М4,75 т11,2 км/с
Vulcan-Centaur 564 (2020)$260M ГПО dV=1800 м/с
Delta IV Heavy (2015)$389M14,220тГПО dV=1800 м/с
1569909596353.jpg112 Кб, 600x418
246 517707
Почему во вселенной существуют пустоты?
247 517710
>>17307
Сразу же ничего не будет.
Через два месяца найдешь сублимированный сухофрукт.
Под пережаренной каменной корочкой будет хуита которую можешь использовать для приготовления компота.
248 517711
>>17707
ТАК ВЫШЛО.
Наука не отвечает на вопросы "почему" да "зачему", это предполагает наличие стремления, цели, какого-то замысла.
Замысла нет.
249 517712
>>17711
Толстишь
250 517714
>>17712
Но он прав. Поиск Великих Замыслов - это удел людей с религиозным типом мышления.
251 517720
>>17714
Каких замыслов?
Слово "почему" не обязательно подразумевает "боженька сделол)))". А причину формирования например, может из-за гравитации войды появились или ещё чего. Я не знаю, поэтому и спрашиваю.
>>17711

> это предполагает наличие стремления, цели,


А познание не цель? Стремление узнать больше о войдах и их формировании - религиозный тип мышления?

Короче, кто о чем, а лысый о расческе. Если вы в таком обычном вопросе видите Бога, разумные замыслы и т.п., то у вас проблемы.
252 517722
>>17663
Нет. Время везде одинаково.
253 517723
>>17230

>это когда кто-то остроумно пёрнул в лужу


Это называется гипотеза. Теория это когда ты берешь гипотезу и развиваешь ее, добавляешь деталей, матана, подгоняешь под известные эксперименты.
254 517725
>>17315
Да хоть три: Земля шарообразна, Солнце светит, эфир существует.
255 517726
>>16766
Тепловая энергия берется из окружающего вакуума из-за неравномерностей в процессах диссипации нуклонов и прочего вещества.
256 517730
>>17230
Теория может быть хорошо проработана, но быть ошибочной - рано или поздно споткнется на некотором эксперименте.
257 517735
>>17725
С первым пунктом ты обосрался, остальные можно не рассматривать. Это если вообще реагировать на твой вброс хоть сколько-нибудь серьёзно.
258 517736
>>17723
Почитай определение и не коверкай его такими словами, пожалуйста.
259 517737
>>17730
Тем не менее, ничего более достоверного, чем научная теория, человечеству неизвестно.
260 517764
>>17707
Следы от квантовых флуктуаций, когда вселенная была пиздюшной, и на нее действовали эффекты квантового мира.
261 517826
>>14000 (OP)
мм да уж
262 517885
если вся материя быыла сгустком меньше атома, то я происхожу от всего? можно сказать что я включаю в себе всю вселенную, или то что вся вселенная включает меня?
263 517893
>>17885
Нет конечно.
265 517914
>>17663
>>17667
В смысле? Разве не наоборот?
266 517916
>>17914
Наоборот, конечно, чем глубже в колодец, тем медленнее идет время. Хуй знает, кто отвечает, что быстрее.
267 517917
В фильме "К Звездам" сцена с крышкой антенны это отсылка к Касcини?
268 517920
Поисните, почему в 100% случаев пиздят о "Вселенной, сжатой в точку" до БВ? Если Вселенная конечная (конечный объем, точнее), то тут ок, вопросов нет. Но если она бесконечна (насколько я знаю, етот вопрос не решен), то как может что-то конечное превратиться в бесконечное? Как же соображения непрерывности и все такое?
EFfWVXRXUAMZtiB1.jpg141 Кб, 1200x900
269 517924
Маск реально собрал ракету в чистом поле и она должна там же и стартовать? Всякие сверхчистые цеха заводов не нужны?
270 517928
>>17924
Это не ракета, а бак с двигателями для тестов в величину корабля. В космическом корабле должна быть система управления, больше датчиков, электричество для СЖО, радиаторы, солнечные батареи, системы связи и прочие приблуды, которых в этом нету.
271 517935
>>17928
роскос, просто соси и не завидуй
272 517952
>>17920
У нее конечное количество массы. Объем вон увеличивается постоянно.
273 517958
>>17952

>У нее конечное количество массы.


Схуяли? В видимой части конечное, а во всей вообще - с чего ты взял?

>Объем вон увеличивается постоянно.


Эт если он конечный. А если он уже бесконечный, куда ему увеличиваться?
274 517970
>>17667
объясни, плз, почему влияет ускорение?
275 517973
>>17928

>Это не ракета, а бак с двигателями для тестов в величину корабля.


Бак с двигателями, который летает = ракета.

>В космическом корабле должна быть система управления


Она там есть. И датчики есть, иначе нахуй такие тесты?

>>17924
Да. А ты думал, все ракеты в чистых комнатах всегда собирали? Нет, и Фау-2, и Союзы, и Редстоуны всякие собирали в обычных таких цехах. В чистых комнатах всякие спутники и аппаратуру делают. Ну, и сами двигатели.
276 517975
Посмотрел фильм "К звёздам" с Питтом. Разочарован. Не сайфай, не космофентезя, не космоопера и не просто что-то явно космическое, а скорее разговорный фильм и история о человеческих взаимоотношениях в антураже космоса. Если бы история происходило не в будущем "пару дней спустя", а, скажем, лет 200-300 назад и была бы о мореплавателях или также лесоходцах, исследователях гор или пустынь - ничего бы не изменилось. Однако, этот фильм вызвал у меня вопрос: в сцене около Нептуна, который подкрашен таинственного синего цвета с мистическим тусклым свечение, сам газовый гигант нормально так видно. Но как бы мы видели Нептун в реале? Насколько там яркое солнце? Поясните на примере хотя бы лампочки разных ватностей или материалов, чтобы я хотя бы примерно почувстовавал
277 517980
>>17975
Фильм не смотрел, но по твоему описанию все нормально. Освещенность в яркий солнечный день на Земле - около 100 000 люкс, без учета атмосферы - 135 000 (https://ru.wikipedia.org/wiki/Освещённость). Нептун в 30 раз дальше, значит освещенность там в 900 раз меньше. То есть около 1500 люкс. По таблице примеров, это как пасмурный день на Земле, на порядки больше того, когда пропадает различение цвета.

Короче, разгадка в том, что глаз может приспосабливаться к освещенности и нормально работать в очень широком диапазоне.
278 517986
>>17975

>Насколько там яркое солнце?



Солнечная постоянная на орбите Земли 1360 Вт/м2.
Солнечная постоянная на орбите Нептуна 2 Вт/м2.
ХЗБ.gif940 Кб, 627x502
279 517987
Если звезда или планета с глючным коллайдером схлопнется в ЧД, заметный глазу аккреционный диск образуется сразу же (из окрестной пыли, например) или понадобится время на сбор вещества?
280 517989
>>17987
Науке это неизвестно.
281 517994
>>17989
Печяльненько, чо.
282 517997
>>17986

>Солнечная постоянная на орбите Нептуна 2 Вт/м2.


Мне это ни о чём не говорит, объясни нормально - лампочками, свечками, спичками хотя бы
283 518001
>>17997
Солнце там в 700 раз слабее светит чем на Земле.
284 518005
1) Орбиты первых четёрх планет вокруг солнца более лимения круглые и ровные, по крайне мере по сравнению с Плутом уж точно - орбиты друг другу не пересекают, лежат в одной плоскости и всё такое. Но насолько они круглые? Вдруг орбита, скажем Земли такова, что в апогей очень близко подбирается к орбите Марса, а перегей - к орбите Венеры, и что Марс и Венера вращаются ровно по кругу, а Земля - овалом? Земля это пример, вопрос ей не ограничен

2) Как создать наиболее эффективный по охлажденю зонд для вращения на низкой орбите (много ниже орбиты Меркурия), чтобы исследовал солнце, но сам не сгорал настолько долго, насколько это возможно?

3) Как проводить исследования Венеры? Допустим, советский венероход, показавший миру единственные фоточкм голой венерианской писечки, на самом деле просто неудачно сел не туда - в зону кратера супервулкна, просто вулкана, или равнины битуумных гейзеров, и что Венера на самом деле куда более приятная и близкая Земле температурой у поверхности (что конечно, мало вероятно), и тогда стоит другим венероходам сесть в другом месте им будет не так жарко, что, однако, никак не отменяет ебического давления и плотных облаков. Как же тогда епердвать радио или какие они там используют сигнлы наверх, в космос, Земле? Как нам обустроить двач обследовать Венеру?
285 518008
>>18005

>Но насолько они круглые?


https://ru.wikipedia.org/wiki/Эксцентриситет_орбиты

>Как создать наиболее эффективный по охлажденю зонд


Дать денег инженерам, чтобы они создали.

>Как же тогда епердвать радио или какие они там используют сигнлы наверх, в космос, Земле?



В каком смысле как?
286 518009
>>17986

>Вт/м2


В данном случае (солнечный свет) сойдет, но вообще эта хуйня мало о чем говорит. Будь там хоть мульон ватт на квадрат радиоволн, например, их ты не увидишь.
287 518010
>>18009

>но вообще эта хуйня мало о чем говорит. Будь там хоть мульон ватт на квадрат радиоволн



На на видимый свет приходится 46% энергии излучаемой Солнцем.
На инфракрасное изучение 47%.
Ну УФ 7%.
288 518011
>>18010

>В данном случае (солнечный свет) сойдет

289 518012
>>18011

>В данном случае


А мы о данном случае (Солнце) тут и говорим, а не о гипотетических звездах.
290 518014
>>17924
Её из силосной башни прямо на ферме в ракету переделали?
291 518016
>>18005
3) Венера вся примерно одинаковой температуры из-за того, что ее плотная атмосфера крайне эффективно переносит тепло. Такая уж у нее охуительная газовая оболочка, что там нет ни климатических поясов, ни сезонов, ни даже смены дня и ночи (освещенность тоже не меняется, всегда одинаковый сумрак).

Передавать сигналы будут так же, как передавали их советские зонды — через орбитальный ретранслятор. Прямиком до Земли пробиться сигналу слишком тяжело, да и энергии на это уйдет слишком много.
292 518017
>>18016

>Венера вся примерно одинаковой температуры из-за того, что ее плотная атмосфера крайне эффективно переносит тепло.



Наоборот. Она такая горячая потому, что ее атмосфера крайне эффективно не пропускает тепло. Луркай "Парниковый эффект".
293 518018
>>18017
Про конвекцию и скорость ветра на Венере слышал?
294 518019
>>18018

>Про конвекцию и скорость ветра на Венере слышал?


И как скорость ветра не Венере противоречит сказанному мною?
295 518020
>>18019
Никак. Только вот ты стал писать про "наоборот" в ответ на пост, который расписывал одинаковость температуры.
296 518021
>>18020

>Только вот ты стал писать про "наоборот"


Но ведь я все верно написал. Тот анон ошибся утверждая, что "атмосфера Венеры крайне эффективно переносит тепло". Она украине плохо его переносит, потому что почти целиком состоит из углекислого газа, поэтому там так жарко.
297 518022
>>18021
И как парниковость газа влияет на его способность переносить тепло?
298 518023
>>17973

>Фау-2, и Союзы, и Редстоуны всякие собирали в обычных таких цехах.



Но хотя бы в цехах, а не на открытом воздухе же.
299 518024
>>18022

>И как парниковость газа влияет на его способность переносить тепло?


Почитай статью в вики о парниковом эффекте.
300 518025
>>18024
Ясно.
301 518028
>>18021
Подумай, через что лучше передается тепло, через вакуум или через газ? Через разреженный газ или через плотный? Через плотный газ или через твердое тело? Простейшие физические принципы говорят нам, что чем больше давление газа, тем выше его теплопроводность, поэтому передача тепла на Венере идет в десятки раз более эффективно, чем на Земле.

Под толщей атмосферы у самой поверхности все температурные различия уже сглажены, и различий в температуре реально почти нет.

>>18024
Это такой троллинг тупостью? Парниковый эффект не имеет абсолютно никакого отношения к теплопроводности, и вызывается он исключительно разницей в прозрачности атмосферы для солнечного спектра и для лучей инфракрасного диапазона.

Видимый свет (с длинами волн, соответствующих температуре Солнца) легко проникает глубоко внутрь атмосферы сквозь прозрачный для него углекислый газ и поглощается поверхностью, но обратно излучается спектр с температурой уже не 5800К, а всего 700К, т.е. гораздо более длинные волны. Для них толща углекислого газа все равно что глухая стена, и просто так улететь в космос они уже не могут, вместо этого постоянно поглощаясь и переизлучаясь молекулами газа и поднимая его температуру.
302 518030
>>18028

>Подумай, через что лучше передается тепло, через вакуум или через газ?



А какая связь между тем, что в вакууме нет передачи тепла при помощи теплопроводности и конвекции и тем фактом, что на Венере высокая температура из-за парникового эффекта вызывного тем, что основу атмосферы Венеры составляет парниковый газ СО2.
303 518032
>>18028

>обратно излучается спектр с температурой уже не 5800К, а всего 700К, т.е. гораздо более длинные волны. Для них толща углекислого газа все равно что глухая стена, и просто так улететь в космос они уже не могут



Следовательно вот это утверждение неверно

>плотная атмосфера крайне эффективно переносит тепло

304 518034
>>18030
Никакой, на это и указывали. Высокая температура от парникового газа и равномерная температура от высокой плотности не связаны вообще никак.

Если бы на Венере была атмосфера не из углекислого газа, а из, например, прозрачного для инфракрасных волн азота, эффективная передача тепла бы все равно была, как и вызванное ей равенство температуры по всей планете, а парникового эффекта бы не было; я вообще хуй знает, зачем ты его приплел, когда мы говорили только про теплопроводность.
305 518035
>>18032
Это натуральный нон секвитур, ты просто хуйню несешь уже. Не вижу смысла дальше участвовать в этом троллинге тупостью.
306 518036
>>18034

>Никакой, на это и указывали



Указывали на другое, а именно на то, что атмосфера Венеры плохо пропускает тепло.
307 518042
>>18034

>Если бы на Венере была атмосфера не из углекислого газа


Начались виляния жопой, кхех.
308 518045
>>17920
Если посмотреть на плоскость с торца, она будет выглядеть линией. Линия с торца выглядит точкой. Точка не имеет размера. Это тебе пример из математики, как бесконечное может оказаться конечным.
309 518046
>>18045

>Если посмотреть на плоскость с торца


Невозможно если плоскость бесконечная. :3
310 518047
>>18032
Ты слышал когда-нибудь фразу одно другому не мешает?
Речь шла не о том, почему на Венере тепло, а о том, почему на Венере равномерно тепло.
311 518048
>>18046
Вот, пространство вокруг тебя бесконечно. Твое существование в нем невозможно?
312 518050
>>18048

>Вот, пространство вокруг тебя бесконечно


Так.

>Твое существование в нем невозможно?


Не вижу никакой логической связи с твоим первым утверждением.

Ты лучше расскажи как собрался смотреть на бесконечную плоть с торца.
313 518051
>>18050

>на бесконечную плоскость с торца.


fix
314 518057
>>18005

>1) Орбиты


Гугли орбиты планет, куча есть картинок и анимаций.

>2) Как создать наиболее эффективный по охлажденю зонд


Паркер.

>3) Как проводить исследования Венеры? Допустим


Мы знаем, что Венера вся такая, для этого не нужно садиться в разных местах. Алсо, это не венероход был, потому что он никуда не ходил, это был просто лендер. А так, сейчас много есть концептов исследования Венеры, ищи в новостных тредах, венеротреде, АМС треде, там постили.
315 518061
Почему Макс делает свой новый корабль на улице? Ещё и варит стыки сваркой. Это ещё что за пиздец?
316 518069
>>18061
Потому что может. Зачем всё переусложнять?
317 518073
>>18069
Оно у него вообще полетит?
318 518082
Из чего состоят чёрные дыры? Пиздос.
319 518093
>>18082
Вопрос "из чего состоят" неявно подразумевает, что объект описывается квантовополевой теорией с иерархией масштабов. Для черных дыр это не так, для их описания на "микроскопическом уровне" (скажем, из которого можно получить статистическое описание их термодинамики) требуется гипотетическая квантовая теория гравитации, которая пока не известна. Известно, что это не будет квантовая теория поля (по крайней мере, с конечным количеством видов полей).
320 518094
>>18073
Через пару месяцев сам увидишь. Его более мелкий Стархоппер уже летал на одном таком двигателе, теперь пришла очередь башни на трех движках.
321 518095
Когда люди исследуют гравитацию и научатся ей управлять это будет прорыв покруче колеса или двигателя.
322 518096
Максимально тупые вопросы ITT.

1) Как так вышло, что белые люди проебали б-жественное наследие фон Брауна и ушли с головой в Спейс Шатл, которые угандошил дохуя огурцов и оказался дорогущей хуйней без задач, да еще и оставил токсичное говно "SLS" после себя? Можно ли сказать, что совочек спасло от такого же движения нахуй закрытие Бурана? Может ли Маск повторить эту порочную траекторию со Старшипом, если не будут реализованы цели по его многоразовости/орбитальной дозаправке/ретропропульсивной посадке?

2) Можно ли отснять Венеру в метровом разрешении на низкоорбитальный SAR, сохранить охулиарды байт данных на массив из накопителей и отправить его на Землящку в спускаемом аппарате с SEP, вместо того, чтобы ебаться с передачей этой инфы по радио? Где подводные камни? Большая масса возвращаемого модуля? Недостаточная защита накопителей от радиации?
323 518098
>>18096

>шатол


поначалу идея казалось хорошей, а когда поняли что не очень - было поздно что-то менять. плюс изначально шатол планировался в комплексе с многочисленными орбитальными станциями денег не дали, в такой ситуации без него хуево было бы

>совок


учитывая что у них хватило мозгов бездумно скопировать шатол - это просто случайность что в ссср/рф удалось сохранить пилотируемые полеты

>муск


хуй знает. общая концепция на бумаге хороша, но по мере разработки возникает много проблем. от способа решения этих проблем зависит будет ли старшип говном или нет

>венера


размеры и вес аппарата прежде всего, радар в том числе. ну и защита от радиации тоже
но прежде всего это не слишком целесообразно сейчас
в теории можно конечно
324 518100
>>18098

> это просто случайность что в ссср/рф удалось сохранить пилотируемые полеты


Деньги на Буран удачно закончились до того момента, как успели выпилить Союзы.
>>18096

> Можно ли отснять Венеру в метровом разрешении на низкоорбитальный SAR, сохранить охулиарды байт данных на массив из накопителей и отправить его на Землящку в спускаемом аппарате с SEP, вместо того, чтобы ебаться с передачей этой инфы по радио? Где подводные камни? Большая масса возвращаемого модуля? Недостаточная защита накопителей от радиации?


Полагаю, передать это по радио будет все же быстрее и дешевле, чем ебаться с возвращаемыми аппаратами.
325 518105
>>18100

>Полагаю


В пдфке по EnVision по передаче всё совсем печально.
326 518169
>>18046
Ну на полуплоскость, например, можно, а она тоже бесконечная. Но я тут один хуй не вижу связи с вопросом о "соображениях непрерывности". Если функция имеет конечное значение при t < t0, а при t = t0 - бесконечное, то это разрыв. При этом нарушаются все законы сохранения, например.

Вот, кстати, что нашел в педивикии:

>The Big Bang was "small": It is misleading to visualize the Big Bang by comparing its size to everyday objects. When the size of the universe at Big Bang is described, it refers to the size of the observable universe, and not the entire universe.[129]



То есть, в принципе, это не противоречит ситуации со Вселенной бесконечного объема.

Кстати, в той статье нигде не упоминается размер сингулярности (конечный или бесконечный): говорится только о плотности вещества, которая стремится к бесконечности в точке сингулярности.
327 518215
>>18100

>Деньги на Буран удачно закончились до того момента, как успели выпилить Союзы.


Буран и не предполагался в качестве основного корабля, им должен был стать ТКС. Из одного из них модуль "Заря" потом сделали.
328 518216
>>18215

> основного корабля, им должен был стать ТКС.


А зарю зачем тогда разрабатывали?
ТКС в пилотируемом варианте был чисто для военных, и стоил столько дохуя что никакой заменой союзу быть не мог.
329 518217
>>18215
хуйню несешь
ТКС дебильная концепция, хуй знает зачем каждый раз возить с собой модуль станции
К тому же модуль называется ФГБ, и из него практически все станции салют и мир
>>18216
Потому что ТКС говно
330 518221
>>18096

>1) Как так вышло, что белые люди проебали б-жественное наследие фон Брауна и ушли с головой в Спейс Шатл, которые угандошил дохуя огурцов


>и оказался дорогущей хуйней без задач


Ключевое слово тут - оказался. При чем во многом по чисто организационно-политическим причинам. Шаттл это часть программы STS, а она в свою очередь инженерный проект "дороги фон-Брауна". Суть: делаем шаттлами орбитальную станцию, орбитальную заправку, пилим орбитальные буксиры, все выводим шаттлами. За эту концепцию так или иначе держались вплоть до конца шаттла - и концепция оказалась провальной.

Однако главный секрет шаттла это то что он не шаттл те челнок. Это короткоживущая орбитальная станция. Тогда правда его цена перестает быть огромной и становится вполне разумной на фоне полученных результатов.

> Можно ли сказать, что совочек спасло от такого же движения нахуй закрытие Бурана


Нельзя. Буран изначально придумывался исходя из того что американцы всё врут на тему своего STS (не может солдат столько брюквы сьесть). Буран изначально делался сверхдорогой иобой без задач, из расчета что когда наша разведка узнает секрет шаттла - вот тогда задачи и появятся. То есть если американцы обосрались, советы просто насрали себе в штаны потому что американцы обосрались, а советы решили что американцы обосрались специально и это хитрый план.
331 518223
>>18221
Это как ты решил что концепция провальная если сначала на станции вообще денег не давали, а дали только на фридом/МКС через лет 25, в которой не было и нет никакой орбитальной инфраструктуры вообще?
332 518234
>>18223

>Это как ты решил что концепция провальная если сначала на станции вообще денег не давали, а дали только на фридом/МКС через лет 25, в которой не было и нет никакой орбитальной инфраструктуры вообще?


Решил не я и решил результатам МКС и шаттла
а) сделать челнок, то есть постоянно многоразово дешево на орбиту грузовик не удалось. По выводам комиссии Орбитер так и остался опасным экспериментальным аппаратом с неустранимыми проблемами.
б) сборка чего-то на орбите оказалась адом, и сейчас главный результат МКС это "избегать орбитальной сборки когда возможно".
Пример: сколько времени роисские космонавты делали задачу "пара фотак и соскоб в месте дырки на союзе"? Пару часов? А остальные задачи ВКД - это же смех один! То что на земле потные мексы делают на несколько минут - в космосе это сначала годами подготовка на земле, потом часы работы с травмами: Как я понял, значительная часть микротравм приходится на кисть и пальцы из-за того что приходится продавливать пальцами скафандр под давлением - в результате за время долгих ВКД особо удачливые успевают стереть кожу до мяса, заработать гематомы на пальцах и добиться отслоения ногтей.
333 518235
>>18223

> в которой не было и нет никакой орбитальной инфраструктуры вообще


И кстати ее там по этому и нет. МКС то здоровая, хотя бы экспериментально орбитальную инфраструктуру там могли сделать. Но делать не стали, и планировать не планируют.
334 518237
>>18234

>а)


зачем ты выдаешь проблемы реализации за неверность концепции?

>б)


сможешь привести список что на МКС специально сделано для работы вне станции? А на скафандре? Ниче там нет кроме поручний и шлюза, и на скафандрах аналогично нет никаких конструктивных особенностей для работы
Так зачем ты опять выдаешь фактически отсутствие внятной реализации на ошибочность концепции?
335 518240
>>18095
Ага.
336 518245
>>18237

>>а)


>>б)


>зачем ты выдаешь проблемы реализации за неверность концепции?


Потому что эту концепцию предлагают для реализации в реальном мире а не в воображении любителей фентази. Подразумевается что концепция так придумана, что осуществима в реальном мире сейчас. За разумные сроки, с разумными технологиями и разумными деньгами.

Если это не так - это неверная концепция.

> для работы вне станции? А на скафандре? Ниче там нет кроме поручний и шлюза, и на скафандрах аналогично нет никаких конструктивных особенностей для работы



Факты: С момента первого выхода в космос прошло более полувека (54 года). За это время было придумано и проведено множество экспериментов в области орбитальной сборки. Придумано и сделано куча инструментов, методик для работ в космосе в невесомости - при чем придумано умнейшими людьми планеты. Тем не менее то что на земле занимает минуты, в космосе занимает часы, приносит травмы, несет риск для жизни и требует многогодовой подготовке к конкретной миссии на земле.

> Ниче там нет


Ну то есть результат полувековой работы человечества по ВКД - ниче там нет. Пиздец.

Мой тезис - если полвека пытались и получился хуй, значит это блять сложна. А концепции которые считают что это проста - неверные и идут нахуй.

Твой тезис какой? ЗОГ не дает привинтить на МКС волшебные вундервафли превращающий ВКД с орбитальной сборкой в легкую прогулку?
337 518249
>>15798

>взрыв, как в ДВС


Маня, иди падмойса, в ДВС именно что горение смеси. Детонация (именно взрыв) убивает двигло к хуям.
338 518261
>>18245

>Потому что эту концепцию предлагают для реализации в реальном мире а не в воображении любителей фентази. Подразумевается что концепция так придумана, что осуществима в реальном мире сейчас. За разумные сроки, с разумными технологиями и разумными деньгами.


ну давай расскажи что там за технологические ограничения не позволяют сделать дешевый и безопасный шатол

>За это время было придумано и проведено множество экспериментов в области орбитальной сборки.


ссылки на эксперименты

>Придумано и сделано куча инструментов


летающее кресло с шаттла и космический шуруповерт?

>Тем не менее то что на земле занимает минуты, в космосе занимает часы, приносит травмы, несет риск для жизни и требует многогодовой подготовке к конкретной миссии на земле.


и все это связано с банальным неудобством, ну кроме рисков
по решению проблем удобства особых попыток не было
а раз не было попыток - нельзя говорить о провальности концепции
про подготовку вообще пиздец, зачем ты выдаешь общую подготовку за подготовку к конретной ВКД?

>Ну то есть результат полувековой работы человечества по ВКД - ниче там нет. Пиздец.


Работы ВКД начались только с шаттлов в 90х, хаббл, мкс и прочее. И кстати нормально там все делалось, цеплялись на канадарм и охуенно было, опора есть, не надо лазить по поручням и все такое.

>полвека пытались


Еще раз - никто не пытался 50 лет наладить орбитальную сборку, все что было и есть - это выходы на небольшой ремонт

>Твой тезис какой?


Такой что орбитальная сборка сама по себе подразумевает что собираем мы что-то большое, дорогое и сложное. А так как на это нет денег - не особо целесообразно практиковать орбитальную сборку. Почитай про концепт сборки JIMO например
339 518264
>>18050
Блядь, ну пусть это будет полуплоскость. Это принципиально?
340 518268

> ну давай расскажи что там за технологические ограничения не позволяют сделать дешевый и безопасный шатол


С чего ты взял что обязательно технологические? Чувак, мы в реальности а не в твоем воображении - если ограничения не технологические это по прежнему невозможность.

>ссылки на эксперименты


> по решению проблем удобства особых попыток не было


> а раз не было попыток - нельзя говорить о провальности концепции


> вы все врети


Пофиксил, не благодари.

> летающее кресло с шаттла и космический шуруповерт?


Что тебя смущает? Что нет волшебных единорогов, а раз нет, значит нещитово?

> Еще раз - никто не пытался 50 лет наладить орбитальную сборку, все что было и есть - это выходы на небольшой ремонт


Мы не можем сделать что-то простое, это оказалось сложно. Почему? Потому что мы не пробовали делать что-то очень большое и сложное! Очень большое и сложное обязательно получится, в отличие от простого.
Логика уровня бог.

> Такой что орбитальная сборка сама по себе подразумевает что собираем мы что-то большое, дорогое и сложное.


Нет. Есть фундаментальный принцип от простого к сложному. Ты его отрицаешь?
Сначала воткнуть кабель и закрутить гайку, потом сборка JIMO, потом сборка из мелких простых элементов всей конструкции. Если на уровне "воткнуть кабель" у нас пиздец, значит концепция не работает.
341 518275
>>18268

>нет ответа про заявленные тобой ограничения


>нет ответа про заявленные тобой эксперименты


>нет ответа про заявленные тобой особые инструменты



>Логика уровня бог.


>Если на уровне "воткнуть кабель" у нас пиздец, значит концепция не работает.


твой пример это примерно если мне надо починить кран в ванной, я чето там руками покрутил, неудобно было пиздец, руки натер как на ВКД, но вроде работает. а все потому что у меня нет разводного ключа
Понел теперь?

Я кстати тоже могу орать что вывод грузов целиком это тупиковый путь, ну там ограничения по диаметру, солнечне панели и радиаторы не влезут а вручную раскрывать же западло, так допиздимся что космонавтика вообще не нужна а может ты к этому и идешь так как являешься МКС-хейтером
342 518276
Что такое звуковая волна и чем она принципиально отличается от радиоволны? Почему скорость радиоволны в воздухе примерно = скорости света, а скорость звука примерно 300 м/с? Разве это не чит передавать звук на огромные расстояния радиоволнами?
343 518277
>>18276
звук - колебания вещества
радиоволна - поток фотонов
344 518284
>>18276
Звук распространяется в какой-то среде (материи), находящейся в пространстве. Частицы среды (молекулы газов воздуха, скажем) то кучкуются погуще, то разлетаются поширше. Если рассмотреть какую-то выделенную точку, то в ней будет колебание плотности среды. Как волна, это колебание распространяется во все стороны, с угасанием. Отсюда, кстати, следует, что звуковая волна не может быть произвольной мощности: отрицательной плотности не бывает, да и для огромной положительной банально среды вокруг не хватит.

Свет распространяется в светоносном эфире электромагнитном поле. Грубо говоря, в каждой точке пространства есть несколько величин, характеризующие электрические и магнитные силы, действующие на тела в этой точке. Световая волна, она же фотон, в любой момент находится в одной точке (хотя квантмех загадочная щтука, и все точки получаются "размытыми"), распространяется по прямой, а не во все стороны. Колебания заключаются в том, что прирост тех самых величин поля (в точке, где фотон вот прямо щас) дрыгает туда-сюда.

Как-то так вроде.
345 518287
>>18234

>Как я понял, значительная часть микротравм приходится на кисть и пальцы из-за того что приходится продавливать пальцами скафандр под давлением - в результате за время долгих ВКД особо удачливые успевают стереть кожу до мяса, заработать гематомы на пальцах и добиться отслоения ногтей.


Ты заебал постить свою же пасту так, как будто это заявление какого-то авторитетного человека.
1475845234382.png229 Кб, 825x1340
346 518292
>>18284

>Свет распространяется в электромагнитном поле


Нет. Свет порождает электромагнитное поле. Фотон это переносчик электромагнитного взаимодействия подобно тому, как звуковая волна - переносчик звукового давления. И вот это "облако" виртуальных фотонов, скачущих между электрическими зарядами, и образует поле.

А вот в какой среде распространяется свет - сие тайна великая есть. Считается, что "просто так", а светоносного эфира не существует. Его действительно не существует, если подходить к вопросу в ньютоновском смысле, считая, что эфир это такой газ.
Но у нас есть т.н. физический вакуум, который нихуя не пустой, умеет куда-то растягиваться, имеет конечную скорость передачи взаимодействий и вообще ведёт себя как-то не как пустота. И здесь следует поставить ребром вопрос об элементарности элементарных частиц.

Вот адроны и лептоны это материя. Из них можно слепить материальную среду и гонять по ней звук либо фазовые состояния, создавая бегучие и стоячие волны. Фонон, магнон, ротон и.т.п. это квазичастицы, из которых состоит "постматерия". Они сами по себе никак не влияют на количество элементарных частиц, существуя как бы "поверх" них, но ведут себя подобно элементарным частицам.

Тогда логично предположить, что если существует постматерия, то может существовать и предматерия. Такое нечто, для которого наши элементарные частицы являются квазичастицами. Брана из М-теории описывает как раз нечто подобное. И вот если светоносный эфир существует, то сделан он из предматерии. Т.е. на нашем "этаже" он нефизичен, но порождения его физичны.
347 518295
>>18264
Половина бесконечной плоскости тоже какбы бесконечна.
>>18276

>Разве это не чит передавать звук на огромные расстояния радиоволнами?


Товарищ Маркони, поздравляю с подключением.
348 518297
>>18275

>>нет ответа про заявленные тобой ограничения


>>нет ответа про заявленные тобой эксперименты


>>нет ответа про заявленные тобой особые инструменты


Ты не обосновал ни одного своего утверждения ничем кроме как неявного "я верю". Почему я тебе должен чтото там постить? При этом ты споришь с очевидными вещами - человеки в космосе с ВКД 54 года, но нужны ДОКАЗАТЕЛЬСТВА что люди учились делать орбитальные сборки. Пиздец.


https://en.wikipedia.org/wiki/EASE/ACCESS

https://www.nasa.gov/centers/johnson/pdf/584726main_Wings-ch3e-pgs130-156.pdf
Куча указаний на разработанные методы и инструменты. Да, волшебных единорогов нет.

NASA ran a series of tests beginning with a deployable solar power wing experiment on Discovery’s first flight
(Space Transportation System [STS]-41D in 1984) to validate the construction techniques that would be
used to build the ISS.

These shuttle crew members were trained extensively for their respective missions

>твой пример это примерно если мне надо починить кран в ванной, я чето там руками покрутил, неудобно было пиздец, руки натер как на ВКД, но вроде работает. а все потому что у меня нет разводного ключа



Оставаясь в пределах твоей аналогии: разводной ключ пока не только не изобрели, но даже непонятно что это такое, в попытках изобрести две суперждержавы потратили полвека и триллионы долларов и непонятно возможен ли этот "разводной ключ" на современном техническом уровне. Все что изобрели за полвека и триллионы долларов - это завинчивать руками.
И тут ты такой - нужно потратить еще триллионы долларов и десяток лет времени точно всё будет зуб даю.

Это и есть пиздец, концепция не работает.

>Я кстати тоже могу орать что вывод грузов целиком это тупиковый путь, ну там ограничения по диаметру, солнечне панели и радиаторы не влезут а вручную раскрывать же западло


Я раз что ты не поехал крышей совсем как тут некоторые.

Идея в том что в коридоре "вывод грузов целиком" тупик дальше. Основную коммерческую и научную деятельность ведут аппараты целиком собранные на земле.

> так допиздимся что космонавтика вообще не нужна


Пилотируемая космонавтика вот для этого собственно и нужна. Что бы поехавшим "ряяя счас настроим цилиндров онейла" тыкать прямо носом в то что ничего мы не можем. И для того что бы не пропустить момент когда сможем.
348 518297
>>18275

>>нет ответа про заявленные тобой ограничения


>>нет ответа про заявленные тобой эксперименты


>>нет ответа про заявленные тобой особые инструменты


Ты не обосновал ни одного своего утверждения ничем кроме как неявного "я верю". Почему я тебе должен чтото там постить? При этом ты споришь с очевидными вещами - человеки в космосе с ВКД 54 года, но нужны ДОКАЗАТЕЛЬСТВА что люди учились делать орбитальные сборки. Пиздец.


https://en.wikipedia.org/wiki/EASE/ACCESS

https://www.nasa.gov/centers/johnson/pdf/584726main_Wings-ch3e-pgs130-156.pdf
Куча указаний на разработанные методы и инструменты. Да, волшебных единорогов нет.

NASA ran a series of tests beginning with a deployable solar power wing experiment on Discovery’s first flight
(Space Transportation System [STS]-41D in 1984) to validate the construction techniques that would be
used to build the ISS.

These shuttle crew members were trained extensively for their respective missions

>твой пример это примерно если мне надо починить кран в ванной, я чето там руками покрутил, неудобно было пиздец, руки натер как на ВКД, но вроде работает. а все потому что у меня нет разводного ключа



Оставаясь в пределах твоей аналогии: разводной ключ пока не только не изобрели, но даже непонятно что это такое, в попытках изобрести две суперждержавы потратили полвека и триллионы долларов и непонятно возможен ли этот "разводной ключ" на современном техническом уровне. Все что изобрели за полвека и триллионы долларов - это завинчивать руками.
И тут ты такой - нужно потратить еще триллионы долларов и десяток лет времени точно всё будет зуб даю.

Это и есть пиздец, концепция не работает.

>Я кстати тоже могу орать что вывод грузов целиком это тупиковый путь, ну там ограничения по диаметру, солнечне панели и радиаторы не влезут а вручную раскрывать же западло


Я раз что ты не поехал крышей совсем как тут некоторые.

Идея в том что в коридоре "вывод грузов целиком" тупик дальше. Основную коммерческую и научную деятельность ведут аппараты целиком собранные на земле.

> так допиздимся что космонавтика вообще не нужна


Пилотируемая космонавтика вот для этого собственно и нужна. Что бы поехавшим "ряяя счас настроим цилиндров онейла" тыкать прямо носом в то что ничего мы не можем. И для того что бы не пропустить момент когда сможем.
349 518298
>>18287

>Ты заебал постить свою же пасту так, как будто это заявление какого-то авторитетного человека.


Маня, это не не моя паста. Это цитата анона которую я превратил в пасту. Вот ему стыдно наверное хахаха. Там еще ссылка на твиттор с доказательствами. Ее я пощу когда очередной дебил орет ряяяяя ниверю.
350 518309
>>18297
Бляяяя

>ссылки


ты их хоть читал сам? это как раз успешные проекты
причем их совсем немного
а помнишь как ты пару постов назад задвигал про огромное количество экспериментов и инструментов? дай угадаю, ты пиздаболил даже не глядя в гугл?

>в попытках изобрести две суперждержавы потратили полвека и триллионы долларов


ясно, то есть ты посчитал абсолютно все расходы на пилотируемую космонавтику как затраты на исследования орбитальной сборки
но вот незадача, сам ты привел лишь пару экспериментов и универсальный канадарм, который не только для ВКД годится

>Идея в том что в коридоре "вывод грузов целиком" тупик дальше


нет, он уже достигнут практически - 10м диаметр на слс и 7-9 на старшипе

>Что бы поехавшим "ряяя счас настроим цилиндров онейла" тыкать прямо носом в то что ничего мы не можем.


А что надо сделать чтобы смочь? Наверно нарочито искажать факты против ВКД-работ и топить за исключительно целые аппараты?
А может все таки стоит дальше развивать ВКД, ну например поставить пару канадармов чтобы покрывать всю МКС, что-нибудь придумать с перчатками скафандров?
351 518361
>>18295

>Товарищ Маркони, поздравляю с подключением.



Я может ошибаюсь, но разве господин Маркони первый запилил радиотелефонию? Он же вроде телеграфом передал, ключом?
352 518366
Может ли так быть, что жизнь в космосе существует в молекулярных облаках в виде полициклических ароматических углеводородов?
353 518367
Все спят что ли?
354 518368
Пол-седьмого уже, пора просыпаться.
355 518369
>>18368
Ещё пять минут
356 518370
>>18295

> Половина бесконечной плоскости тоже какбы бесконечна.


Торец-то есть, есть куда смотреть.
>>18045
357 518371
>>18370

>Торец-то есть, есть куда смотреть.


Как может быть торец у бесконечной плоскости? Ты идешь его ищешь, а его нет, ведь она бесконечна.
358 518373
>>18277
Какой размер фотона у радиоволн низкой частоты?
359 518375
>>18371
У бесконечной ленты есть целых джва торца
360 518376
>>18375

>У бесконечной есть


Если что-то где-то заканчивается, значит оно не бесконечно.
361 518418
>>18309

> ты их хоть читал сам?


Я то да. А вот ты ни их ни чего другого не читал, я внезапно понял.
И именно по этому требовал от меня ссылок.

> ряяяя пара экспериментов пиздаболил немного незадача



Я кажется понял в чем твоя проблема. Ты считаешь что орбитальная сборка это такая волшебная эльфийская хрень для отработки которой нужны волшебные обряды. И по этому нет этой сборки - обряды не проведены, злые силы мешают.

Задам простой вопрос: вот первый выход в открытый космос это отработка орбитальной сборки? По твоему "хахаха конечно нет. это же не волшебный обряд N2". В реальности же орбитальная сборка состоит на 99% из простых операций "выйти из шлюха", "добраться до места проведения работ", "сунуть болт в отверстие" и так далее. Безо всякого волшебного обряда N2 типа "сварка в вакууме" или волшебного "ох какое волшебство завинтить болт" которые занимают 1%.

И жопа в том что эти базовые примитивные операции так и остались адом. Опасными для жизни (не шутка), рутинно травмирующими экипаж, пиздец стрессом, очень дорогими по баблу, требующие годов подготовки на земле(ссылку я привел), и атски потребляющими время. И все ради того что бы закрутить пару гаек.

То есть все эти ужасы и миллиарды что бы решить проблему решаемую за 15 минут на земле мексом за постабаксов.

> стоит дальше развивать ВКД


Стоит конечно развивать, что за вопрос. И развивают. Но надеятся и малафьей заливать, как ты, уже нет. Всё, капут. Концепция оказалась неудачной. Удачная концепция хуйнуть готовый хабитат и посещать его по мере необходимости. А когда ресурс кончится либо затопить либо спустить вниз старшипом.

А вот когда будет совсем другая концепция и технологии - например телеуправляемые роботы, тогда может быть. И под будет имеется в виду что БУДЕТ. То есть робот возьмет болт и сунет его куда надо как мекс на земле, за пару секунд. А не "мы примотали к федору скотчем дрель ряяя".

> ну например поставить пару канадармов чтобы покрывать всю МКС


И ты считаешь что это уберет тот ад которым является орбитальная сборка?
И ты считаешь что НАСА это такие дебилы которые за ~35 лет не догадались?

>, что-нибудь придумать с перчатками скафандров?


Проблема стоит полвека. Она была ясна сразу после первого ВКД.
За полвека пока чего-то ничего не придумали.
И тут ты такой "ряяя счас все порешаем"
Не порешаешь.

>искажать факты против ВКД-работ


Чувак, ты уже прокололся что по вопросу ничего не читал кроме фантастики. Ты даже pdf по ссылке не читал.

> нет, он уже достигнут практически - 10м диаметр на слс и 7-9 на старшипе


Более узкие диаметры осваивали полвека. Ну вот 9 метров старшипа тоже будут полвека.
А еще есть надкалиберные обтекатели, 18 метров муска, и главное - надувные хабитаты.

Еще раз, ты рассуждаешь с позиций фентази. "Диаметр +100 метров иначе несчитово, нет розовых единорогов". Я рассуждаю с позиций реализма - плюс 5метров это пиздец дохуя.

> ты посчитал абсолютно все расходы на пилотируемую космонавтику как затраты на исследования орбитальной сборки


Не все, а например половину. Или даже треть. Каждый ВКД это исследование орбитальной сборки - рутинное повторение экспериментов 60-х.
361 518418
>>18309

> ты их хоть читал сам?


Я то да. А вот ты ни их ни чего другого не читал, я внезапно понял.
И именно по этому требовал от меня ссылок.

> ряяяя пара экспериментов пиздаболил немного незадача



Я кажется понял в чем твоя проблема. Ты считаешь что орбитальная сборка это такая волшебная эльфийская хрень для отработки которой нужны волшебные обряды. И по этому нет этой сборки - обряды не проведены, злые силы мешают.

Задам простой вопрос: вот первый выход в открытый космос это отработка орбитальной сборки? По твоему "хахаха конечно нет. это же не волшебный обряд N2". В реальности же орбитальная сборка состоит на 99% из простых операций "выйти из шлюха", "добраться до места проведения работ", "сунуть болт в отверстие" и так далее. Безо всякого волшебного обряда N2 типа "сварка в вакууме" или волшебного "ох какое волшебство завинтить болт" которые занимают 1%.

И жопа в том что эти базовые примитивные операции так и остались адом. Опасными для жизни (не шутка), рутинно травмирующими экипаж, пиздец стрессом, очень дорогими по баблу, требующие годов подготовки на земле(ссылку я привел), и атски потребляющими время. И все ради того что бы закрутить пару гаек.

То есть все эти ужасы и миллиарды что бы решить проблему решаемую за 15 минут на земле мексом за постабаксов.

> стоит дальше развивать ВКД


Стоит конечно развивать, что за вопрос. И развивают. Но надеятся и малафьей заливать, как ты, уже нет. Всё, капут. Концепция оказалась неудачной. Удачная концепция хуйнуть готовый хабитат и посещать его по мере необходимости. А когда ресурс кончится либо затопить либо спустить вниз старшипом.

А вот когда будет совсем другая концепция и технологии - например телеуправляемые роботы, тогда может быть. И под будет имеется в виду что БУДЕТ. То есть робот возьмет болт и сунет его куда надо как мекс на земле, за пару секунд. А не "мы примотали к федору скотчем дрель ряяя".

> ну например поставить пару канадармов чтобы покрывать всю МКС


И ты считаешь что это уберет тот ад которым является орбитальная сборка?
И ты считаешь что НАСА это такие дебилы которые за ~35 лет не догадались?

>, что-нибудь придумать с перчатками скафандров?


Проблема стоит полвека. Она была ясна сразу после первого ВКД.
За полвека пока чего-то ничего не придумали.
И тут ты такой "ряяя счас все порешаем"
Не порешаешь.

>искажать факты против ВКД-работ


Чувак, ты уже прокололся что по вопросу ничего не читал кроме фантастики. Ты даже pdf по ссылке не читал.

> нет, он уже достигнут практически - 10м диаметр на слс и 7-9 на старшипе


Более узкие диаметры осваивали полвека. Ну вот 9 метров старшипа тоже будут полвека.
А еще есть надкалиберные обтекатели, 18 метров муска, и главное - надувные хабитаты.

Еще раз, ты рассуждаешь с позиций фентази. "Диаметр +100 метров иначе несчитово, нет розовых единорогов". Я рассуждаю с позиций реализма - плюс 5метров это пиздец дохуя.

> ты посчитал абсолютно все расходы на пилотируемую космонавтику как затраты на исследования орбитальной сборки


Не все, а например половину. Или даже треть. Каждый ВКД это исследование орбитальной сборки - рутинное повторение экспериментов 60-х.
362 518429
>>18376
Множество натуральных чисел тоже заканчивается "в одну сторону".
363 518437
>>18261

>Работы ВКД начались только с шаттлов в 90х, хаббл, мкс и прочее.


Ващет работы во время ВКД начались еще на Skylab в семидесятых. Станцию неоднократно чинили снаружи, включая установку двух тепловых экранов.
364 518440
>>18373
С твой писюн
365 518441
>>18437

>Ващет работы во время ВКД начались еще на Skylab в семидесятых. Станцию неоднократно чинили снаружи, включая установку двух тепловых экранов.


Я же говорил что он ничего про ВКД не знает же. Ну вот - еще одно доказательство.
366 518442
Пусть два фотона одной частоты одновременно выпущены из одной точки в одном направлении, с одинаковой поляризацией, но строго в противофазе. Как я понимаю, интерференция должна быть такая, что их какбе и нет вообще. Можно ли такую ситуацию отличить от того, как если бы их вообще не было? Или такого быть не могет?
367 518444
>>18309

> 10м диаметр на слс и 7-9 на старшипе



Кстати:

https://en.wikipedia.org/wiki/BA_2100

BA_2100 Pressurised volume:2250 m3
ISS Pressurised volume:915 m3

Для производственных нужд хватит с головой.
368 518446
>>18440
Ты не понимаешь природу интерференции фотонов. Фотон это не волна, а частица. Поэтому интерференция у них это не то же самое что интерференция волн на воде. Как летели два фотона, так и будут лететь, а если ебанут о атом то возбудят его.
369 518454
Что это всё значит? Можно просто объяснить совсем для тупых (меня)? https://youtu.be/xejyeBOHArE
370 518463
Читали науку интерстеллар Кипа Торна?
371 518466
>>18454
Вот тут пиздато проясняется: https://habr.com/ru/post/468381/

Если кратко, то это скорее доказательство многомировой интерпретации квантмеха, нежели отправка сообщений в прошлое (хотя здесь тонкая грань, ибо многомировая интерпретация сама по себе подразумевает возможность путешествий в прошлое не совсем тривиальным образом).

Т.е. частицы могут иметь несколько интерференций одновременно, которые могут работать вразжопицу, чем херят всевозможное теории скрытых параметров. Здесь мы получили на экране один горб из частиц, якобы потерявших свою волновую функцию. Но если извлечь данные из вспомогательных электронов - обнаружится, что горб на самом деле это две интерференции, наложившиеся друг на друга и скрывающие друг друга. Сам факт прочтения этих данных меняет результат эксперимента - мы видим скрытую интерференцию, которая на самом деле произошла.

В многомировой интерпретации всех этих парадоксов нет. Мы сначала неявно-отсекаем одну группу параллельных реальностей (сохранив в кванте информацию о том, что именно отсекли), затем обратно склеиваем, затем отсекаем другую группу и получаем обособленную реальность. Но когда мы извлекаем из кванта информацию - обнаруживается, что это не обособленная реальность, а две, у которых интерференция перекрывает друг друга.
372 518477
>>18466
Уф...Спасибо, стало гораздо понятней. А как тогда быть с квантовы бессмертием? Возможно ли оно?
373 518483
>>18477
Нет
374 518488
>>18217

>ТКС говно


А не спеть ли м пойти ли тебе нахуй?
ТКС охуенен и опередил время.
Если бы не кококогептилофобия была бы охуенная машина до сих пор.
375 518490
>>18488
ну давай расскажи нахуй он нужен если ты не собираешь станцию мир на каждом полете
376 518506
>>18371
Написал же, полуплоскость.
>>18376
Что насчёт луча скажешь?
377 518513
Какой уровень радиации в космосе? А то я чет гуглю и нет никакой информации.
378 518521
>>18513
Где именно в космосе? Он довольно большой.
379 518540
>>18521
За орбитой луны и до Марса.
380 518549
>>18540
Из https://en.wikipedia.org/wiki/Health_threat_from_cosmic_rays

>Astronauts on Apollo and Skylab missions received on average 1.2 mSv/day and 1.4 mSv/day respectively.[20]


>Estimates are that humans unshielded in interplanetary space would receive annually roughly 400 to 900 mSv (compared to 2.4 mSv on Earth) and that a Mars mission (12 months in flight and 18 months on Mars) might expose shielded astronauts to roughly 500 to 1000 mSv.[28]


Короче, в районе 1-3 миллизивертов в сутки обещают. Хз правда, что насчет солнечных вспышек.
381 518550
>>18549
Спасибо.
D1ZaMW0AEqruF.jpglarge226 Кб, 2048x1152
382 518553
Что это за хуёвины?
383 518557
Почитал немного про ракету Pluto, у которой рабочее тело- забортный воздух, и появились некоторые мысли. Что, если сделать орбитальный самолёт с подобной системой движения? Что-то вроде шаттла, только с воздухозаборником на брюхе, с большими баками для воздуха внутри корпуса и силовой установкой, способной нагревать этот воздух для достижения нужной тяги. Выводится этот аппарат с помощью РН на орбиту, поработал там- рабочее тело заканчивается. Тогда орбитальный самолёт делает нырок в атмосферу( что-то вроде нырка шаттла, которым пугали КПСС), наполняет баки воздухом и снова выходит на орбиту. Цикл повторяется.
Если вышеперечисленное верно, то можно создать целую систему электромагнитных ускорителей/ малых ракет, которые выводили бы грузы за атмосферу, но не на орбиту, а данный аппарат "подхватывал" бы их и выводил бы на орбиту. Помимо этого таким аппаратом можно было бы ловить космический мусор и инспектировать/корректировать орбиты спутников.
384 518558
>>18513

>Какой уровень радиации в космосе?



Разный.
385 518559
>>18557
А неплохая идея. На Земле вряд ли получится обеспечить требуемую холодопроизводительность для сжижения азота/кислорода, но на Марсе и Венере углекислый газ можно сжижить просто сжатием хотя тоже охлаждать придется.
sage 386 518560
>>18557
эта хуйня работает примерно до 5-8 махов. для орбиты нужно 25.
387 518561
>>18557

>наполняет баки воздухом и снова выходит на орбиту



А за счет чего он снова должен выйти на орбиту? Для этого ему придется запустить свой ядерный двигатель и засрать атмосферу радиацией.
388 518562
>>18558
Насколько разный? При вспышках какой будет уровень?
389 518564
>>18562

>Насколько разный?


Если ты на низкой орбите вокруг Земли, под зашитой ее магнитного поля, как МКС, то за год ты получишь дозу около 200 миллизиверт. Если ты поднимешься чуть выше и попадешь в радиационные пояса, то там уже будут десятки миллизиверт в ЧАС. Выйдя за пределы радиационных поясов будет примерно в 2 раза выше, чем на МКС.

Если говорить о Солнечной системе цвелом, то чем ближе ты к Солнцу, тем выше плотность потока электромагнитного излучения и заряженных частиц. Чем дальше от Солнца тем ниже плотность потока электромагнитного излучения и заряженных частиц от Солнца, но вы плотность потока заряженных частиц приходящих из внешних источников.
390 518596
>>18442
Энергия-то никуда не девается, анон.
391 518598
>>18490
Но я собираю станцию мир на каждом полете. Проблемы?
392 518600
>>18596
Куда девается энергия единичных фотонов, когда они годами путешествуют между звёзд, и испытывают красное смещение при расширении вселенной в то время, как они преодолевают расстояние от звезды до наблюдателя? Поскольку в пути длина их волны увеличивается, а значит, уменьшается энергия.
393 518602
>>18600
В гравитацию, вот куда. Летит такой фотон и воздействует на черную дыру в центре галактики.
spacesuit.jpg108 Кб, 625x484
394 518613
>>18287
Запоздало, но вдруг кому нибудь понадобится.
Вот, например, целая работа этому посвящена
https://www.ncbi.nlm.nih.gov/pubmed/20922881

>A significant number of astronauts sustain hand injuries during extravehicular activity training and operations. These hand injuries have been known to cause fingernail delamination (onycholysis) that requires medical intervention. This study investigated correlations between the anthropometrics of the hand and susceptibility to injury.


Пик - табличка из статьи. В среднем ~9,5% риск травмы руки при ВКД.
395 518619
>>18602
кек
396 518635
>>18560
Это в низких слоях атмосферы. В разряженной атмосфере и тем более в вакууме тяга должна быть больше
397 518636
>>18635
это как больше если рабочего тела все меньше?
398 518642
>>18636
Так и сопротивление атмосферы меньше, а рабочее тело будет "зачерпываться" в более плотных слоях.
399 518646
>>18600
Просто исчезает в никуда, ни в какую потенциальную энергию она не уходит.

Закон сохранения энергии это ж не настоящий закон, а просто следствие симметрии физических процессов относительно обращения времени. Если обратить время и получить сокращающуюся вселенную, то фотоны будут приобретать энергию из ниоткуда.
401 518654
>>18600
Рассеивается в эфире.
>>18602
Вот этот в первом предложении верно говорит. Энергия уходит в общее гравитационное поле. Ведь напряженность гравитационного поля это по сути температура эфира. Фотон стареет, рассеивает вокруг свою энергию - окружающий эфир чутка греется - общее гравитационное поле подпитывается. Потом когда-нибудь рассеянная этим фотоном энергия будет участвовать в приталкивании вещественных тел друг к другу.
402 518657
>>18652
А закон сохранения энергии работает для процесса расширения вселенной?
403 518658
>>18654

>Энергия уходит


Мозги у тебя походу уже ушли.
404 518673
>>18652
Если прочитаешь эту самую статью, то выяснишь, что это не фундаментальный закон, а просто следствие теоремы Нётер (симметрии времени), которая работает только локально и в плоском пространстве.

Если можешь в английский, то неплохо расписано в FAQ в https://www.reddit.com/r/askscience/wiki/astronomy — пункты:

>Universe expansion - Does the expanding universe conserve energy?


>Universe expansion - Why doesn't the redshifting of photons by expansion violate the conservation of energy?

405 518692
>>18673

>Закон сохранения энергии не фундаментальный закон


Ну все, теперь тебе осталось только написать в Нобелевский комитет и премия у тебя в руках
406 518706
>>18682 (Del)
>>18692
А вот и закономерные подрывы спелеоматериалистических пидорашек подъехали, нойс.
407 518715
>>18477

>Уф...Спасибо, стало гораздо понятней. А как тогда быть с квантовы бессмертием? Возможно ли оно?


Возможно, разумеется. Однако, экспериментально проверить его не-субъективно - ОЧЕНЬ нетривиальная задача.

Мы можем взять, например, выборку больных с какими-то тяжёлыми пороками сердца, которым впоследствии сделали пересадку сердца или клапана. До пересадки они могли откинуть копыта от любого стресса, а потом всё стало норм. И мы обнаружим, что до пересадки в жизни им несказанно везло, а после в их жизни стало больше неудач, ведь как только от их груди убрали "квантовый пистолет" - они стали способны пронаблюдать эти неудачи. Однако для постороннего наблюдателя всё это - не более, чем ошибка выжившего.

Можно залезть в большой ящик с тысячей человек и атомной бомбой и тыкать там на кнопку, ловя осечки. Так можно было бы пронаблюдать групповое квантовое бессмертие, но внешним людям ты тем самым не докажешь ничего.

По-настоящему годные эксперименты в этой области начнутся тогда, когда появятся методы редактирования памяти индивида. В этом случае можно будет обойтись без убийств и самоубийств.
408 518726
Почему частица становится в супер позицию?
409 518728
На каком этапе сейчас ЯРД? Что и кем сейчас разрабатывается и на каком оно этапе? В чем основные сложности ядерки, что до сих пор нет ничего практичного?
410 518729
>>18728
на этапе "в принципе сделать можно, но решение сложностей нецелесообразно"
разработок нет уже лет 30, недавно НАСА заявила о интересе
сложности в материалах турбины которые выдерживали бы нужные температуры и общем отводе тепла
нет ничего практичного потому что сами по себе они не имеют смысла, нужны только как часть программ по серьезному освоению космоса, а на это денег не дают после лунных полетов
411 518761
>>17528

>https://twitter.com/CaliaDomenico/status/1178748703638740993


Битлджус реально так бахнет? Блин, так хочется чтобы свет от этого события до нас дошёл пока я живой и дееспособный...
412 518767
>>18761
И что ты будешь по этому поводу деять?
413 518772
>>18767
Смотреть@охуевать
414 518780
>>18729
То есть выход — осваивать космос на химии до тех пор, пока он не станет достаточно важным для политиков, чтобы начать финансировать ядерного буксира?
415 518781
>>18780
Ну, можешь основать глобальную корпорацию по производству носков, заработать миллиарды, а потом их вложить в разработку ЯРД.
416 518791
>>18780
ага ага опять политики мешают
но обосновать экономическую целесообразность с цифрами ты конечно же не сможешь, только невнятные крики про науку и прогресс
417 518796
>>18791
А какая экономическая целесообразность проёбанного на F-35 триллиона долларов? Неужели дополнительные 10 миллиардов из бездонного военного бюджета на буксир ради НОУКИ это так много? Может НОУКА и ПРОГРЕСС не нужны? Что сказать то хотел?
418 518799
>>18796
Повышение обороноспособности страны конечно же.
10 миллиардов кстати хватит только на телескоп от производителя Ф-35 лол

>на буксир ради НОУКИ


Какой НОУКИ? Что ты будешь возить на нем, компоненты лунной базы, мощные межпланетные зонды? Но они тоже денег стоят, причем дохуя, и этих денег как раз нет. А нет денег на такие проекты - нахуй нужен буксир если на нем нечего возить?
419 518823
>>18799
10 миллиардов в год, алё. За 10 лет это почти стоимость Апполона и вполне реальный срок для разработки пихла. Причём оборонный бюджет даже не почувствует. На разработку лунной базы можно направить остальной бюджет насы с СЛС и аналогичного бесполезного.
420 518824
>>18823
Да и 10 миллионов в год на одно пихло — это оверкилл. Явно меньше денег потребуется.
421 518826
>>18824

>Явно


Бля, вот как можно заранее знать, сколько тебе потребуется, если нет технологии, материалов, и так далее? Ладно построить небоскреб - все технологии есть, материалы известные. Сложить, умножить, посчитать. А тут как?
422 518829
>>18553
TTGL посмотри
423 518832
>>18799

>Повышение обороноспособности страны конечно же.


За триллион баксов можно было бы 33 тысячи (!!!) F-15E наклепать и задоминировать к хуям собачьим всю планету. О какой обороноспособности идёт речь вообще? Вся эта программа - один сплошной военно-политический и экономический саботаж.
424 518874
>>18832

>3 тысячи (!!!) F-15E наклепать и задоминировать к хуям собачьим всю планету.


Где ты столько летчиков возьмешь? Что бы подготовить одного летчика нужно несколько миллионов долларов.
425 518876
>>18874

>Где ты столько летчиков возьмешь?


На сдачу с этой сверхгигантской суммы. Стоимость лётчика это в любом случае 5-10% от стоимость самолёта, не больше.
426 518878
>>14000 (OP)
Откуда на Земле дохуя воды?
427 518880
>>18878
Кислород крайне распространение в космосе, а водород вообще самый распространенный элемент. В космосе полно воды.
EuropasOceanKPHand003.jpg599 Кб, 1024x768
428 518887
>>18878
Да не то, что бы сильно дохуя.
429 518892
>>18878
От вулканизма. В мантии ее дохуя.
430 518893
>>18887
Бля, а ведь по Европе можно будет плавать насквозь, как в звездных войнах по планете Набу.
431 518905
>>18893
Нет, нельзя.
432 518906
>>18878
Кометы принесли.
433 518947
Янепонил, кварковые и преонные звёзды мыслятся спрятанными под горизонтом событий или как? Ведь у нейтронной звезды почти уже предельно минимальный радиус, а у кварковой - на порядок меньше. Значит, это по сути попытка описания структуры чёрной дыры?
434 518949
>>18947

>Янепонил, кварковые и преонные звёзды мыслятся спрятанными под горизонтом событий или как?



Видимо да. Но ты пойми, что это лишь гипотезы, продукт размышлений теоретиков, у них задача - придумывать новые модели которые бы не противоречили общей теории относительности.
435 518950
>>18906
В капусте принесли?
436 518969
>>18887
Это с учетом подземных вод?
437 518973
Почему вокруг Земли нету мелких каменных спутников? Почему система Плутон-Харон сохранила мелкие луны вроде Гидры и Стикса, а Земля и Луна раскидали всё к хуям?
438 518985
>>18969
У Европы - объем воды при глубине океана в 100 километров при разбросе оценки в 70-180 км. То есть вероятно ее больше, но с меньшей вероятностью может быть и несколько меньше.
У Земли - вся вода начиная с грунтовых вод и выше.

>This sphere includes all of the water in the oceans, ice caps, lakes, rivers, groundwater, atmospheric water, and even the water in you, your dog, and your tomato plant


Количество воды в мантии, насколько я понимаю, не учитывается, поскольку оно неизвестно.
439 519003
>>18973
с земной Луной, вообще, дохренища неясного, и появление, и отношение масс, и период вращения, и состав.
440 519004
>>18950
В решете.
441 519013
>>18905
Угу, как скажешь
442 519056
>>18973

> а Земля и Луна раскидали всё к хуям?


Скорее всего это раскидала Юпитер и Сатурн когда были гораздо ближе к Солнцу.
443 519097
>>18557

>данный аппарат "подхватывал" бы их


чтобы он не разбился об "похватываемые" грузы, эти грузы надо разогнать до орбитальной скорости, а если они уже разогнаны до орбитальной скорости, то нахрен этот аппарат тогда нужен?
444 519174
>>17528
Эта хуйня уже взорвалась, скорее всего. Свет никак дойти только не может.
10682902-3x2-700x467.jpg51 Кб, 700x467
445 519175
Наверняка уже обсуждалось, но все же - а что если построить на орбите большой рельсотрон и запускать из него АМС или корабли в вакууме? Какие скорости будут? Или и людям, и технике пиздец будет от моментального ускорения? А все-таки - какие скорости? Можно как то с пользой это применить?
446 519180
>>19175
Если на орбите, то сам рельсотрон будет всякий раз улетать в другую сторону, лучше на Луне. Заодно можно в кратер поместить, где Солнца нет. Ускорение можно сделать мягче, скоростей можно добиваться очень больших. Можно использовать, например, для доставки продукции, произведённой на Луне, на Землю, или для вывода компонент космических кораблей, построенных на Луне. Но такая штука может потенциально стать серьёзным оружием.
447 519185
>>19180

>потенциально стать серьёзным оружием.


И может, и станет. Импульс можно и ионниками постепенно погасить. А так да, шмалядло которое пихает тонны со скоростью больше 4км/с это не только пушка лучше имеющихся на вооружении, но и двигатель даже неплохой.
448 519186
>>19175
Рельсотроны и на Земле хуево работают, требуя гигантского количества энергии и частой замены рельс, которые уже после пяти-десяти выстрелов приходят в полную непригодность. В данный момент нет нужных материалов, чтобы рельсотроны были чем-то практичным.
449 519188
>>19175
объясни целесообразность и экономическую выгоду
450 519193
>>19188
Ты дурак? Он об этом же и спрашивает. Ты на любой вопрос отвечаешь заданием этого же вопроса?
1400322399634.jpg48 Кб, 800x450
451 519197
Почему видима неосвещенная сторона Луны?
Это как вообще блядь?
Я что, вижу отраженный свет звезд? Отраженный от Земли свет?
Или лунная поверхность сама излучает в видимом спектре?
Что это за хуйня вообще блядь, думаю уже 3-й день как это вообще возможно
452 519199
>>19197

>Отраженный от Земли свет?


Это.
453 519213
>>14328
какие технологии.
-во-первых российский робот Федор ананцефал. У него нет мозга и вообще он не робот , а аватар, теле-зонд. в РФ никаких роботов вообще нет, для этого нужны всякие си программисты с нейронными сетями, а в РФ таких людей нет.
-во вторых даже если рассматривать не роботов, а управляемые теле-зонды типа Федора, напомню что он не может даже удерживать в руке предмет, ему приклеивают предмет к руке изолентой левые люди.
-немецкий Саймон уже поумнее, но для безопасности у него нет рук, чтоб он вдруг не построил третий рейх.
-американцы запускают всяких полуроботов, типа марсоходов, но это капля в море
454 519214
>>18832
может в сша решили прикормить впк фирмы. а может они просчитали, что для достижения превосходства в текущей ситуации необходимы малозаметные ф35, а не 33000 ф15.

может у них нет авианосцев или аэродромов в ключевых местах, под 33 000. может они считают что эти 33 000 бесполезны, так как будут эффективно сбиваться пво, и не смогут выполнить сша-доктрину(разнести пво, а потом господствуя в воздухе разнести фронты)
455 519219
>>18893
Ты ебнутый,лол? У нее ядро есть,такая же железяка как у земли.
456 519228
Помню читал, что американцы рейлганом разогнали болванку до 7 км/с. Вроде бы это первая космическая. Значит можно огромной рельсой разгонять космический аппарат хотя бы до условных 5 км/с, а довыводить на орбиту уже собственными двигателями космолета?
457 519229
>>19228
ускорение слишком высокое будет, никакие механизмы не выдержат
а для норм ускорения нужна пушка в километры длинной
458 519233
>>19228
Твоя болванка сгорит уже через секунду, а пепел затормозится и упадет.

На уровне моря с космическими скоростями летать вообще нельзя, слишком сильно растет сопротивление (квадратично от скорости) и нагрев (от кубической зависимости от скорости для теплопроводности до восьмой степени для излучения).

>>19229
Километры это ты еще мало взял, даже если со 100g ускоряться, то процесс разгона займет 7 секунд, а пройденный путь составит 24,5 км. Если запускать людей, то там ну самый предел это 10g, и пушка вырастет еще в 10 раз, до 245 километров.
459 519235
Действительно ли земля может стать такой же как венера ?
460 519236
>>19233

>Твоя болванка сгорит уже через секунду, а пепел затормозится и упадет.


Так у них ничего не сгорало, они даже по мишеням стреляли.
image.png1,6 Мб, 1260x910
461 519237
>>19236
ну хуй знает
462 519239
463 519240
>>19236
Неси ссылку, в которой рейлган осилил 7 км/с, да еще и по мишеням. Подозреваю, что по факту окажется, что и не в лотерею, а в преферанс, и не выиграл, а проиграл.
464 519241
>>19235
Нет, у нас ни такого количества углерода нет, ни Солнце так сильно не печет в данный момент, через пару миллиардов лет другой разговор будет. На градусов 20 от современной температуры еще можно поднять, но венерианского климата на Земле не будет.
465 519242
>>19219
Схуяли железяка? Ледышка.
466 519244
>>19242
Схуяли ледышка то? Каким образом? От приливного захвата бы давно расстаяла,ядро как минимум каменное должно быть.
image.png2,2 Мб, 1280x960
467 519245
468 519246
Сколько воды надо вылить в космос, чтобы образовался стабильный астероид исключительно из жидкой воды и водяного пара? Температура за счет солнечного излучения ровно 20 градусив, выливали аккуратно, так что гравитационного разогрева нет, вращение мегакапли - оборот за 5 часов, чтобы не было бешеной конвекции. (Под стабильностью я понимаю очень медленную потерю атмосферы, скажем не более 0,0001% за земной год).
469 519253
>>19246
Как минимум сферу радиусом с половину земного, я полагаю. Луна точно слишком маленькая, чтобы удержать жидкую воду в любом количестве, а Марс уже точно достаточно большой (но он гораздо плотнее воды).

Кстати, в центре все равно лёд будет, а не жидкая вода.
470 519254
>>19244
Она частично расстаяла, но вода под давлением превращается в лед, не забывай.
471 519255
>>19253

>Кстати, в центре все равно лёд будет, а не жидкая вода.


Хм, но ведь лед менее плотен чем вода и должен всплывать. Будет ли там бурбулятор из льда и воды околонулевой температуры в центре?

И будут ли недра без собственного источника тепла вообще охлаждаться? Снаружи-то везде теплая вода, да еще и атмосфера из водяного пара должна давать парниковый эффект, нет?
472 519256
>>19255
Не путай то, что по речкам плавает, с твердой водой в целом. Льда куча видов, и те, которые под высоким давлением и температурой выше 0°С существуют, более плотные, чем вода — там кристаллическая решетка более плотно упакована.
473 519257
>>19255
Лед, который под давлением, имеет другую плотность.
474 519258
>>19256
>>19257
Да, судя по тому, что давление в центре Земли на три порядка выше чем то, что нужно для такого льда, и в водном астероиде давления хватит.
475 519261
>>19258
А что, хочешь построить такую планету? Воду лучше налить на Марс.
476 519263
>>19254
>>19245
Вот тебе строение европы дружок
477 519264
>>19261
Ну это будет Земля на минималках, скукота. Вот чисто водяная планета будет чем-то новым. Прилетят инопланетяне - то-то удивятся.

Кстати, вот еще подумал: если есть ядро из плотного холодного льда (3, 9), то на него может намерзать обычный лед-1. Он хоть и менее плотный, чем вода, но всплывать не будет, потому что заякорится за ядро. То есть, планета в итоге будет иметь здоровое ледяное ядро, сверху океан жидкой воды, и потом уже атмосферу из водяного пара. Похоже на правду?
478 519265
>>19263
Чел, это просто картинка.
479 519266
>>19264
Да, так и будет. Но жизни на ней не будет.
480 519267
>>19264
Бляить,вы вообще понимаете,что такого быть в принципе не может?
Или сюда челы из зог протекли
481 519268
>>19266
Ну это понятно. По условиям же только вода (дистиллированная!).
482 519269
>>19267
Считай, что это что-то вроде огурцеводства, только без графона и с более гибкими правилами. (И без огурцов).
483 519271
>>19265
Масса и радиус Европы отлично известны. Отсюда находится плотность, которая в разы больше той, какая была бы от чистой воды. Наверняка при построении "картинки" были использованы и куда более хитрые выводы, но уж этот может любой школьник повторить.
484 519273
>>19265
Это хорошая гипотеза, лучше которой пока нет. Доживи до Европы Клиппера и миссии JUICE, узнаешь получше.
i.jpeg13 Кб, 290x180
485 519275
Задал Пашке вопрос про Вселенную на стриме, он отправил меня в пешее на другой свой канал мистики. Настораживает.
486 519277
>>19275
Тут задавай, чо ты каких-то хохлов смотришь. Я рыцарь ордена Ганимеда, знаю ответы на все вопросы мира.
487 519278
>>19277
Мне сама интерпретация от его дица была интересна, так как он оче умный персонаж.
488 519283
>>19278
От нас ты че хочешь?
489 519290
>>19235
Такой же как Венера стать не может, но теплее стать может. Смотри, разница в глобальных средних температурах между полностью ледниковыми периодами и безледниковыми оценивается примерно в 10º C. Что это значит? Когда жили динозавры средняя температура Земли была выше, чем в настоящее время, хотя Солнце светило слабее.
Сейчас ледниковый период, единственное не ясно заканчивается он или продолжается.
490 519297
>>14000 (OP)
Оснащались ли американские космические корабли системой самоуничтожеения при приземлении на территории враждебного государства?
491 519299
>>19297
Американцы о таком не упоминают. Думаю, что у них такого не было, такие штуки были характерны для параноидального СССР.
492 519301
>>19290
В данный момент на Земле очевидно присутствуют ледниковые щиты и по прогнозам в ближайшие десятки тысяч лет они не растают, так что конец оледенения еще не близко. В нормальное время вне ледниковых периодов на планете вообще фактически не бывает многолетнего льда (только в горах в высоких широтах).
493 519309
>>19301

>и по прогнозам в ближайшие десятки тысяч лет они не растают,


Ну, тут можно долго ломать копья. Одни утверждают, что началось потепление из-за антропогенного фактора другие говорят что никакого потепления нет и антропогенный фактор это хуита.
494 519311
>>19297
Лол, конечно же нет.
495 519312
>>19275
Кто такой? Какую ученую степень имеет и в каком научном центре работает?
496 519313
>>19273
Два чаю, смысла гадать сейчас нет. Надо отправлять АМС и изучать.
497 519314
>>19241

>Нет, у нас ни такого количества углерода нет


А сколько у насс тонн углерода в биосфере?
498 519315
>>19236
Сгорит, точно сгорит. В плотных слоях атмосферы полет с такой скоростью моментально все нахуй испарит.
499 519316
>>19213

>во-первых российский робот Федор ананцефал


Сделал мою ночь.
500 519318
>>19315
Зависит от формы болванки, материала, вращения. Заострённая хуйня войдет как масло
501 519319
>>19175
На орбите смысла нет, ибо отдача. А вот на космических телах не имеющих атмосферы почему бы и нет?
Тред тупых вопросов №114 Zeldovich edition 502 519320
Тред вопросов о жизни, Вселенной и всём таком.

Спрашиваем то, за что в других местах выдают путёвку в биореактор. Здесь анонимные учёные мирового уровня критически рассмотрят любые гениальные идеи и нарисованные в Paint схемы.

Предыдущий тут: >>14000 (OP)
https://2ch.hk/spc/res/514000.html (М)

Q: Можно быстрее?
A: Можно упасть в Пузырь Алькубьерре, NASA уже почти надула его (это шутка).

Q: Я начитался охуительных историй про уфологию, че делать, нам жопа?
A: Да, тебе жопа, можешь сгонять в зогач или куда оттуда пошлют.

Q: Что будет с человеком в вакууме без скафандра / если он упадет на черную дыру / попробует ступить на поверхность газового гиганта/солнца?
A: Он умрёт.

Q: Почему бы не привязать ракету к воздушному шару или стартовать с горы?
A: Космос - это не как высоко, а как быстро, большая часть энергии ракеты уходит на разгон вбок.
Подробнее тут https://what-if.xkcd.com/58/ (английский) https://chtoes.li/orbital-speed/ (перевод)
503 519322
>>19320
гений йопт
504 519325
>>18947
Вроде бы нет. https://www.youtube.com/watch?v=u4RNGRyzt10

>Ведь у нейтронной звезды почти уже предельно минимальный радиус


Да, для ее массы. Кварковые звезды по идее должны быть плотнее, но как ты сам и сказал, они на порядок меньше размером.
505 519354
>>19320
Заебца!
506 519359
>>19320
Ебанный стыд...
Во-первых, Алькубьерре.
Во-вторых, не упасть, а создавать вокруг корабля изнутри (иначе кина не будет).
В-третьих, НАСА искривляет пространство на десятимиллионную часть, контролируя это сверхточными интерферометрами, до самого варп-привода здесь - как до Антарктиды раком.
507 519402
>>19309
Третьи говорят, что дальше на ожидает похолодание.
1 1 508 541701
1
Тред утонул или удален.
Это копия, сохраненная 18 января 2020 года.

Скачать тред: только с превью, с превью и прикрепленными файлами.
Второй вариант может долго скачиваться. Файлы будут только в живых или недавно утонувших тредах. Подробнее

Если вам полезен архив М.Двача, пожертвуйте на оплату сервера.
« /spc/В начало тредаВеб-версияНастройки
/a//b//mu//s//vg/Все доски